Financial Management Reviewer
Financial Management Reviewer
In smaller businesses in which the management of cash is would be $72,000 per year. However, the bank will
but one of numerous functions performed by the treasurer, charge $48,000 (12 months x $4,000 per month) for
various cost incentives and diversification arguments its services. Thus, the firm will increase its income by
suggest that surplus cash should be invested in $24,000 ($72,000 - $48,000).
D. Corporate bonds.
[3] Source: CMA 1280 1-3
Answer (A) is incorrect because a small firm may not The prime lending rate of commercial banks is an
have enough surplus cash to invest in commercial announced rate and is often understated from the viewpoint
paper, which usually consists of secured or unsecured of even the most credit-worthy firms. Which one of the
promissory notes of large corporations. following requirements always results in a higher effective
interest rate?
Answer (B) is incorrect because the transactions cost
of bankers' acceptances is high. A banker's A. A floating rate for the loan period.
acceptance is a unique credit instrument used to
finance both domestic and international B. A covenant that restricts the issuance of any new
"self-liquidating" transactions. It is usually initiated by unsecured bonds during the existence of the loan.
a bank's irrevocable letter of credit on behalf of the
bank's customer, on which the company doing C. The imposition of a compensating balance with an
business with the bank's customer draws a time draft. absolute minimum that cannot be met by current
The company discounts the time draft with the transaction balances.
company's local bank and receives immediate
payment. The local bank forwards the time draft to D. The absence of a charge for any unused portion in
the bank customer for payment. the line of credit.
Answer (C) is correct. A small firm with surplus cash Answer (A) is incorrect because the floating interest
should invest for the highest return and lowest risk. rate is not always higher. It should float up or down
The ability to convert the investment into cash without with the prime rate.
a loss of principal is also important. Money market
mutual funds invest in money market certificates such Answer (B) is incorrect because a restriction on a
as treasury bills, negotiable CDs, and commercial new issuance does not raise the interest rate on
paper. Because of diversification, these mutual funds money previously borrowed.
are superior to any single instrument.
Answer (C) is correct. When a firm borrows money
Answer (D) is incorrect because an increase in from the bank, it is often required to keep a certain
interest rates could cause a substantial loss in percentage of the funds in the bank at all times. These
principal. compensating balances effectively increase the rate of
interest on the money borrowed from the bank.
[2] Source: CMA 0683 1-7
A firm has daily cash receipts of $200,000. A commercial Answer (D) is incorrect because, if a firm chooses
bank has offered to reduce the collection time by 3 days. not to use its full line of credit and is not charged for
The bank requires a monthly fee of $4,000 for providing the unused portion, the rate of interest on the portion
this service. If money market rates will average 12% during used does not increase.
the year, the additional annual income (loss) of having the
service is
[4] Source: CMA 0683 1-8
A. $(24,000). A firm's current ratio is currently 1.75 to 1. Management
knows it cannot violate a working capital restriction
B. $24,000. contained in its bond indenture. If the firm's current ratio
falls below 1.5 to 1, technically it will have defaulted. If
C. $66,240. current liabilities are $250 million, the maximum new
commercial paper that can be issued to finance inventory
D. $68,000. expansion is
A. $81,950.
[7] Source: CMA 1283 1-25
B. $83,630. Merkle, Inc. has a temporary need for funds. Management
is trying to decide between not taking discounts from one
C. $96,895. of their three biggest suppliers, or a 14.75% per annum
renewable discount loan from its bank for 3 months. The
D. $99,550. suppliers' terms are as follows:
D. Level of collection expenditures. A. Clay has a low ratio of short-term debt to total
debt while Lott has a high ratio of short-term debt to
Answer (A) is incorrect because the quality of total debt.
accounts is important to credit policy since it is
inversely related to both sales and bad debts. B. Clay has a low current ratio while Lott has a high
current ratio.
Answer (B) is correct. A quantity discount is an
attempt to increase sales by reducing the unit price on C. Clay has less liquidity risk while Lott has more
bulk purchases. It concerns only the price term of an liquidity risk.
agreement, not the credit term, and thus is unrelated
to credit and collection policy. D. Clay's interest charges are lower than Lott's
interest charges.
Answer (C) is incorrect because offering a cash
discount improves cash flow and reduces receivables Answer (A) is incorrect because Clay's aggressive
and the cost of extending credit. policy would result in more short-term debt, with
attendant renewal problems and high risk. Lott's
Answer (D) is incorrect because the level of conservative policy would produce more long-term
collection expenditures must be considered when debt or equity financing.
implementing a collection policy. The marginal cost of
a credit and collection policy should not exceed its Answer (B) is correct. A conservative working
revenue. capital management financing policy uses permanent
capital to finance permanent asset requirements and
also some or all of the firm's seasonal demands. Thus,
[9] Source: CMA 1284 1-20 Lott's current ratio (current assets/current liabilities)
If the average age of inventory is 90 days, the average age will be high since its current liabilities will be relatively
low. An aggressive policy entails financing some fixed CMR's cash balance would increase by $650,000
assets and all the current assets with short-term (6.5 days x $100,000 per day).
capital. This policy results in a lower current ratio.
Answer (B) is incorrect because checks currently
Answer (C) is incorrect because Clay is subject to take 10 ス days to be credited to CMR's cash
greater liquidity risk than Lott since it has greater balance. The lockbox system shortens this process to
short-term debt. Hence, it is at greater risk of being 4 days, a difference of 6 ス days. Since the average
unable to meet its maturing obligations. daily collection is $100,000, a period of 6 ス extra
days yields a $650,000 average increase in the cash
Answer (D) is incorrect because a more conservative balance.
company would tend to finance by means of equity
rather than debt capital. Thus, the more conservative Answer (C) is incorrect because checks currently
company would have less interest expense. take 10 ス days to be credited to CMR's cash
balance. The lockbox system shortens this process to
4 days, a difference of 6 ス days. Since the average
[11] Source: CMA 1285 1-6 daily collection is $100,000, a period of 6 ス extra
An increase in sales resulting from an increased cash days yields a $650,000 average increase in the cash
discount for prompt payment would be expected to cause balance.
A. An increase in the operating cycle. Answer (D) is incorrect because checks currently
take 10 ス days to be credited to CMR's cash
B. An increase in the average collection period. balance. The lockbox system shortens this process to
4 days, a difference of 6 ス days. Since the average
C. A decrease in the cash conversion cycle. daily collection is $100,000, a period of 6 ス extra
days yields a $650,000 average increase in the cash
D. A decrease in purchase discounts taken. balance.
Answer (C) is correct. If the cause of increased sales B. Marketable securities level.
is an increase in the cash discount, it can be inferred
that the additional customers would pay during the C. Proper relationship between current assets and
discount period. Thus, cash would be collected more current liabilities.
quickly than previously and the cash conversion cycle
would be shortened. D. Proper blend of marketable securities and cash.
Answer (D) is incorrect because more customers will Answer (A) is incorrect because credit and collection
take discounts. policies concern receivables and are not influenced
by an EOQ model for inventory management.
[12] Source: CMA 1286 1-30 Answer (B) is incorrect because the level of
CMR is a retail mail order firm that currently uses a central marketable securities is in part determined by cash
collection system that requires all checks to be sent to its needs.
Boston headquarters. An average of 5 days is required for
mailed checks to be received, 4 days for CMR to process Answer (C) is incorrect because the relationship
them and 1 ス days for the checks to clear through its bank. between current assets and current liabilities concerns
A proposed lockbox system would reduce the mail and many factors other than cash management.
process time to 3 days and the check clearing time to 1
day. CMR has an average daily collection of $100,000. If Answer (D) is correct. Since cash and inventory are
CMR should adopt the lockbox system, its average cash both nonearning assets, in principle they may be
balance would increase by treated similarly. The alternative to holding cash,
however, is to hold marketable securities that do earn
A. $650,000. interest or dividends. Thus, a cash management
model would determine how much of a firm's liquidity
B. $250,000. should be held as cash and how much in the form of
marketable securities.
C. $800,000.
[14] Source: CMA 1286 1-32
D. $400,000. According to John Maynard Keynes, the three major
motives for holding cash are for
Answer (A) is correct. Checks are currently tied up
for 10 ス days (5 for mailing, 4 for processing, and A. Transactional, psychological, and social purposes.
1 ス for clearing). If that were reduced to 4 days,
B. Speculative, fiduciary, and transactional purposes. The forms of short-term borrowing that are unsecured
credit are
C. Speculative, social, and precautionary purposes.
A. Floating lien, revolving credit, chattel mortgage,
D. Transactional, precautionary, and speculative and commercial paper.
purposes.
B. Factoring, chattel mortgage, bankers'
Answer (A) is incorrect because the three major acceptances, and line of credit.
motives for holding cash, according to Keynesian
economics, are for transactional, precautionary, and C. Floating lien, chattel mortgage, bankers'
speculative purposes. acceptances, and line of credit.
Answer (B) is incorrect because the three major D. Revolving credit, bankers' acceptances, line of
motives for holding cash, according to Keynesian credit, and commercial paper.
economics, are for transactional, precautionary, and
speculative purposes. Answer (A) is incorrect because a chattel mortgage is
a loan secured by personal property (movable
Answer (C) is incorrect because the three major property such as equipment or livestock). Also, a
motives for holding cash, according to Keynesian floating lien is secured by property, such as inventory,
economics, are for transactional, precautionary, and the composition of which may be constantly changing.
speculative purposes.
Answer (B) is incorrect because a chattel mortgage is
Answer (D) is correct. John Maynard Keynes, a loan secured by personal property (movable
founder of Keynesian economics, concluded that property such as equipment or livestock). Also,
there were three major motives for holding cash: for factoring is a form of financing in which receivables
transactional purposes as a medium of exchange, serve as security.
precautionary purposes, and speculative purposes
(but only during deflationary periods). Answer (C) is incorrect because a chattel mortgage is
a loan secured by personal property (movable
[15] Source: CMA 1286 1-34 property such as equipment or livestock). Also, a
When a company analyzes credit applicants and increases floating lien is secured by property, such as inventory,
the quality of the accounts rejected, the company is the composition of which may be constantly changing.
attempting to
Answer (D) is correct. An unsecured loan is a loan
A. Maximize sales. made by a bank based on credit information about
the borrower and the ability of the borrower to repay
B. Increase bad-debt losses. the obligation. The loan is not secured by collateral,
but is made on the signature of the borrower.
C. Increase the average collection period. Revolving credit, bankers' acceptances, lines of
credit, and commercial paper are all unsecured
D. Maximize profits. means of borrowing.
Answer (A) is incorrect because tightening credit will [17] Source: CMA 0687 1-25
reduce sales and bad debt losses. The carrying costs associated with inventory management
include
Answer (B) is incorrect because tightening credit will
reduce sales and bad debt losses. A. Insurance costs, shipping costs, storage costs, and
obsolescence.
Answer (C) is incorrect because, most likely, higher
quality accounts will mean a shorter average B. Storage costs, handling costs, capital invested, and
collection period. obsolescence.
Answer (D) is correct. Increasing the quality of the C. Purchasing costs, shipping costs, set-up costs, and
accounts rejected means that fewer sales will be quantity discounts lost.
made. The company is therefore not trying to
maximize its sales or increase its bad debt losses. The D. Obsolescence, set-up costs, capital invested, and
objective is to reduce bad debt losses and thereby purchasing costs.
maximize profits. Answer (A) is incorrect because shipping costs are
ordering costs, not carrying costs.
[16] Source: CMA 1286 1-35
The following forms of short-term borrowing are available Answer (B) is correct. Carrying costs include storage
to a firm: costs, handling costs, insurance costs, interest on
capital invested, and obsolescence.
キ Floating lien
キ Factoring Answer (C) is incorrect because it states various
キ Revolving credit ordering (or manufacturing) costs.
キ Chattel mortgages
キ Bankers' acceptances Answer (D) is incorrect because the set-up costs for
キ Lines of credit a production run are equivalent to ordering costs.
キ Commercial paper Additionally, purchasing costs are considered costs
of ordering. as:
[19] Source: CMA 0687 1-29 Answer (D) is incorrect because $54,444, not
(Refers to Fact Pattern #1) $55,056, is the amount of the loan used to determine
The amount Morton Company must borrow to pay the annual interest expense.
supplier within the discount period and cover the
compensating balance is [21] Source: CMA 0688 1-13
A compensating balance
A. $55,000.
A. Compensates a financial institution for services
B. $55,056. rendered by providing it with deposits of funds.
Answer (A) is incorrect because the prime rate has C. Current assets minus inventories to current
nothing to do with a commitment fee on a bank loan. liabilities.
Answer (B) is incorrect because the effective rate on D. Current liabilities to long-term debt.
most companies' bank loans will be much higher than
the prime rate. Answer (A) is incorrect because it does not refer to
expenses that have to be paid out of the sales dollars.
Answer (C) is incorrect because the prime rate is a Also, the sales revenue available to pay liabilities is
bank loan rate, not the rate on commercial paper. indeterminable. Thus, sales minus debt service is not
a liquidity measure.
Answer (D) is correct. The prime interest rate is the
rate charged by commercial banks to their best (the Answer (B) is incorrect because fixed assets are
largest and financially strongest) business customers. included in the numerator of the ratio; hence, this ratio
It is traditionally the lowest rate charged by banks. is not a measure of liquidity.
However, in recent years, banks have been making
loans at still lower rates in response to competition Answer (C) is correct. Liquidity is the degree to
from the commercial paper market. which assets can be converted to cash in the short
run to meet maturing obligations. The usual measures
of liquidity are the current ratio and the quick
[23] Source: CMA 0689 1-10 (acid-test) ratio. The quick ratio is the best measure
Determining the appropriate level of working capital for a of short-term liquidity because it uses only the most
firm requires liquid assets (cash, marketable securities, and
receivables) in the calculation; inventories are not
A. Evaluating the risks associated with various levels included because they are two steps away from cash
of fixed assets and the types of debt used to finance (they have to be sold, and then the receivable has to
these assets. be collected).
B. Changing the capital structure and dividend policy Answer (D) is incorrect because using only liabilities
for the firm. in the calculation ignores the assets available to pay
the liabilities.
C. Maintaining short-term debt at the lowest possible
level because it is ordinarily more expensive than [25] Source: CMA 0689 1-11
long-term debt. Since Marsh, Inc. is experiencing a sharp increase in sales
activity and a steady increase in production, the
D. Offsetting the profitability of current assets and management of Marsh has adopted an aggressive working
current liabilities against the probability of technical capital policy. Therefore, the company's current level of net
insolvency. working capital
Answer (A) is incorrect because management of A. Would most likely be the same as in any other
fixed assets is not a factor in working capital type of business condition as business cycles tend to
management. balance out over time.
Answer (B) is incorrect because capital structure and B. Would most likely be lower than under other
dividend policy are factors involved in capital business conditions in order that the company can
structure finance, not in working capital financial maximize profits while minimizing working capital
management. investment.
following except
C. Would most likely be higher than under other
business conditions so that there will be sufficient A. Decentralized collections.
funds to replenish assets.
B. Electronic funds transfers.
D. Would most likely be higher than under other
business conditions as the company's profits are C. Compensating balances.
increasing.
D. Lockbox systems.
Answer (A) is incorrect because the growing firm is
more apt to emphasize production rather than Answer (A) is incorrect because it is a common
protecting against technical insolvency by maintaining method of accelerating cash collections.
a high level of working capital.
Answer (B) is incorrect because it is a common
Answer (B) is correct. When a firm has an aggressive method of accelerating cash collections.
working capital policy, management keeps the
investment in working capital at a minimum. Thus, a Answer (C) is correct. Various methods of
growing company would want to invest its funds in accelerating cash collections include decentralized
capital goods and not in idle assets. This policy collection outposts (normally one in each Federal
maximizes return on investment at the price of the risk Reserve District), electronic funds transfers,
of minimal liquidity. centralized banking for all company branches to
avoid having to maintain minimum balances in several
Answer (C) is incorrect because the company will locations, and lockbox systems. A compensating
prefer to expend funds on capital goods. balance is a minimum average or absolute amount
that must be maintained in a bank account. Hence, it
Answer (D) is incorrect because the company needs is not a means of accelerating cash collections. This
its profits to invest in new production equipment in requirement means that less cash is available to the
order to grow. depositor.
[26] Source: CMA 0689 1-13 Answer (D) is incorrect because it is a common
Short-term, unsecured promissory notes issued by large method of accelerating cash collections.
firms are known as
[28] Source: CMA 0689 1-14
A. Agency securities. Short-term securities issued by the Federal Housing
B. Interest rates are constant over the short run. B. Common stock.
Answer (D) is correct. The EOQ formula is a Answer (C) is incorrect because it lacks the liquidity
deterministic model that requires a known demand necessary to be a cash substitute. It can also be quite
for inventory or, in this case, the amount of cash a risky investment.
needed. Thus, the cash flow requirements cannot be
random. The model also assumes a given carrying Answer (D) is incorrect because it lacks the liquidity
(interest) cost and a flat transaction cost for necessary to be a cash substitute. It can also be quite
converting marketable securities to cash, regardless a risky investment.
of the amount withdrawn.
[30] Source: CMA 0689 1-17 [32] Source: CMA 1289 1-15
A small retail business would most likely finance its A change in credit policy has caused an increase in sales,
merchandise inventory with an increase in discounts taken, a reduction in the investment
in accounts receivable, and a reduction in the number of
A. Commercial paper. doubtful accounts. Based upon this information, we know
that
B. A terminal warehouse receipt loan.
A. Net profit has increased.
C. A line of credit.
B. The average collection period has decreased.
D. A chattel mortgage.
C. Gross profit has declined.
Answer (A) is incorrect because only large
companies with excellent credit ratings have access to D. The size of the discount offered has decreased.
the commercial paper market.
Answer (A) is incorrect because no statement can be
Answer (B) is incorrect because a retail store must made with respect to profits without knowing costs.
have instant access to its inventory to provide
continuous services to customers. Thus, a loan on a Answer (B) is correct. An increase in discounts taken
terminal warehouse receipt loan would not be suitable accompanied by declines in receivables balances and
because the inventory would not be in the immediate doubtful accounts all indicate that collections on the
possession of the seller. increased sales have been accelerated. Accordingly,
the average collection period must have declined. The
Answer (C) is correct. A small retail store would not average collection period is a ratio calculated by
have access to major capital markets. In fact, the dividing the number of days in a year (365) by the
only options available, outside of owner financing, are receivable turnover. Thus, the higher the turnover, the
bank loans and a line of credit from suppliers. It is shorter the average collection period. The turnover
this latter alternative that is most often used because it increases when either sales (the numerator) increase,
permits the store to finance inventories for 30 to 60 or receivables (the denominator) decrease.
days without incurring interest cost. A line of credit is Accomplishing both higher sales and a lower
an arrangement between a bank and a borrower in receivables increases the turnover and results in a
which the bank commits itself to lend up to a certain shorter collection period.
maximum amount to the borrower in a given period.
Answer (C) is incorrect because no statement can be
Answer (D) is incorrect because a chattel mortgage is made with respect to profits without knowing costs.
Answer (D) is incorrect because the discount may Answer (B) is incorrect because, if the cost of
have been increased, which has led to quicker stockouts declines, the incentive to carry large safety
payments. stocks is reduced.
[33] Source: CMA 1289 1-16 Answer (C) is correct. A company maintains safety
The following information regarding a change in credit stocks to protect itself against the losses caused by
policy was assembled by the Wilson Wax Company. The stockouts. These can take the form of lost sales or
company has a required rate of return of 10% and a lost production time. Safety stock is necessary
variable cost ratio of 60%. because of the variability in lead time and usage rates.
As the variability in lead time increases, a company
Old Credit New Credit will tend to carry larger safety stocks.
Policy Policy
---------- ---------- Answer (D) is incorrect because a decline in the
Sales $3,600,000 $3,960,000 variability of usage makes it easier to plan orders, and
Average collection period 30 days 36 days safety stocks will be less necessary.
The pretax cost of carrying the additional investment in
receivables, using a 360-day year, would be [35] Source: CMA 1289 1-18
The following information regarding inventory policy was
A. $5,760. assembled by the JRJ Corporation. The company uses a
50-week year in all calculations.
B. $9,600.
Sales 10,000 units per year
C. $8,160. Order quantity 2,000 units
Safety stock 1,300 units
D. $960. Lead time 4 weeks
The reorder point is
Answer (A) is correct. The first step is to determine
the average investment in receivables under each A. 3,300 units.
policy. Under the old policy, average daily sales are
$10,000 ($3,600,000/360 days). Given a 30-day B. 2,100 units.
average collection period, the average receivables
balance is $300,000 ($10,000 x 30 days). Under the C. 100 units.
new policy, average daily sales are $11,000
($3,960,000/360 days), and the average receivables D. 1,300 units.
balance is $396,000 ($11,000 x 36 days). Hence,
the average balance is $96,000 higher under the new Answer (A) is incorrect because 3,300 units is the
policy. Because the company's incremental (variable) sum of the order quantity units plus the safety stock in
costs are 60% of sales, the extra investment is only units.
$57,600 (60% x $96,000). The interest rate, or
required rate of return, is 10%. Thus, the incremental Answer (B) is correct. The reorder point is the
carrying cost is $5,760 (10% x $57,600). inventory level at which an order should be placed.
This level is the inventory to be sold during the lead
Answer (B) is incorrect because $9,600 equals the time plus any safety stock. If weekly sales are 200
10% required rate of return times the $96,000 units (10,000/50 weeks) and the lead time is 4
differential between the average receivables balances. weeks, sales during the lead time should be 800 units.
Adding the 800 units of expected sales to the 1,300
Answer (C) is incorrect because the differential units of safety stock produces a reorder point of
between the average receivables balances is 2,100 units.
$96,000, not $136,000.
Answer (C) is incorrect because the reorder point is
Answer (D) is incorrect because $960 assumes a calculated by adding sales during the lead time
10% variable cost ratio. [(10,000/50 weeks) x 4 weeks] to the units of safety
stock.
[34] Source: CMA 1289 1-17
In inventory management, the safety stock will tend to Answer (D) is incorrect because 1,300 units equals
increase if the the units of safety stock required.
[36] Source: CMA 1289 1-20
A. Carrying cost increases. Which one of the following is a spontaneous source of
financing?
B. Cost of running out of stock decreases.
A. Notes payable.
C. Variability of the lead time increases.
B. Long-term debt.
D. Variability of the usage rate decreases.
C. Prepaid interest.
Answer (A) is incorrect because an increase in
inventory carrying costs makes it less economical to D. Trade credit.
carry safety stocks. Thus, safety stocks will be
reduced. Answer (A) is incorrect because it occurs as a result
of transactions apart from purchase transactions. In Answer (B) is incorrect because only large
other words, such credit is arranged separately from companies with good credit ratings can find buyers
the transactions to acquire the assets being financed. for their commercial paper.
Answer (B) is incorrect because it occurs as a result Answer (C) is incorrect because commercial paper is
of transactions apart from purchase transactions. In unsecured.
other words, such credit is arranged separately from
the transactions to acquire the assets being financed. Answer (D) is incorrect because investors must pay a
commission similar to that on other investment
Answer (C) is incorrect because prepaid interest is securities.
not a source of financing.
[39] Source: CMA 1290 1-19
Answer (D) is correct. Trade credit is a spontaneous During the year, Mason Company's current assets
source of financing because it arises automatically as increased by $120, current liabilities decreased by $50,
part of the purchase transaction. and net working capital
A. Increased by $70.
[37] Source: CMA 1289 1-21
The Altmane Corporation was recently quoted terms on a B. Did not change.
commercial bank loan of 7% discounted interest with a
20% compensating balance. The term of the loan is 1 year. C. Decreased by $170.
The effective cost of borrowing is (rounded to the nearest
hundredth) D. Increased by $170.
Answer (C) is incorrect because an aging schedule Answer (A) is incorrect because the annual interest
focuses on uncollected receivables. cost will be 8.66%.
Answer (D) is correct. The purpose of an aging of Answer (B) is correct. The total cost to the company
receivables is to classify receivables by due date. will be $21,200 ($20,000 discount + $1,200 of
Those that are current (not past due) are listed in one transaction costs), and the net amount available will
column, those less than 30 days past due in another be $978,800. The annualized amount of the costs is
column, etc. The amount in each category can then $84,800 (4 x $21,200). Accordingly, the annual
be multiplied by an estimated bad debt percentage interest cost will be 8.66% ($84,800/$978,800).
that is based on a company's credit experience and
other factors. The theory is that the oldest receivables Answer (C) is incorrect because the annual interest
are the least likely to be collectible. Aging the cost will be 8.66%.
receivables and estimating the uncollectible amounts
is one method of arriving at the appropriate balance Answer (D) is incorrect because the annual interest
sheet valuation of the accounts receivable account. cost will be 8.66%.
[42] Source: CMA 1290 1-23 [44] Source: CMA 1290 1-30
As a company becomes more conservative with respect to Lawson Company has the opportunity to increase annual
working capital policy, it would tend to have a(n) sales $100,000 by selling to a new, riskier group of
customers. Based on sales, the uncollectible expense is
A. Increase in the ratio of current liabilities to expected to be 15%, and collection costs will be 5%. The
noncurrent liabilities. company's manufacturing and selling expenses are 70% of
sales, and its effective tax rate is 40%. If Lawson accepts
B. Decrease in the operating cycle. this opportunity, the company's after-tax profit will increase
by
C. Decrease in the quick ratio.
A. $4,000.
D. Increase in the ratio of current assets to
noncurrent assets. B. $6,000.
Answer (B) is incorrect because a decrease in the Answer (A) is incorrect because after-tax income will
normal operating cycle permits a lower level of increase by $6,000.
working capital. If assets can be converted to cash
more quickly, current assets can be reduced. Answer (B) is correct. The company's manufacturing
and selling costs exclusive of bad debts equal 70% of
Answer (C) is incorrect because a decrease in the sales. Hence, the gross profit on the $100,000
quick ratio signifies that quick assets (cash, increase in sales will be $30,000 (30% x $100,000).
receivables, and marketable securities) are Assuming $15,000 of bad debts and $5,000 of
decreasing relative to current liabilities. collection expense, the increase in pre-tax income will
be $10,000 ($30,000 - $20,000). Consequently,
Answer (D) is correct. A conservative working after-tax income will increase by $6,000 [$10,000 -
capital policy results in an increase in working capital (40% x $10,000)].
(current assets - current liabilities). It is typified by a
reduction in liquidity risk. Increasing the current ratio, Answer (C) is incorrect because after-tax income will
whether by decreasing current liabilities or increasing increase by $6,000.
current assets, minimizes the risk that the company
Answer (D) is incorrect because after-tax income will the interest rate is 2.04% for each 20-day period, the
increase by $6,000. annual interest rate (rounded to the nearest tenth) is
36.7% (18 x 2.04%).
[45] Source: CMA 0691 1-4
Which group of ratios would be useful in evaluating the [47] Source: CMA 0691 1-7
effectiveness of working capital management? An organization would usually offer credit terms of 2/10,
net 30 when
A. Profit margin, acid-test ratio, and return on assets.
A. The organization can borrow funds at a rate
B. Acid-test ratio, inventory turnover ratio, and exceeding the annual interest cost.
average collection period ratio.
B. The organization can borrow funds at a rate less
C. Inventory turnover ratio, times interest earned, and than the annual interest cost.
debt-to-equity ratio.
C. The cost of capital approaches the prime rate.
D. Acid-test ratio, current ratio, and return on equity.
D. Most competitors are offering the same terms, and
Answer (A) is incorrect because profit margin and the organization has a shortage of cash.
return on assets are measures of the management of
all assets. Answer (A) is incorrect because, if the company
does not need cash, it would not offer cash discounts,
Answer (B) is correct. Working capital equals regardless of its cost of capital, unless required to
current assets minus current liabilities. The acid-test match competition.
ratio equals quick assets (current assets - inventory -
prepaid expenses) divided by current liabilities. Answer (B) is incorrect because the ability to borrow
Inventory (a current asset) turnover equals cost of at a lower rate is a reason for not offering cash
goods sold divided by average inventory. The discounts.
average collection period (number of days' sales in
accounts receivable, a current asset) equals the Answer (C) is incorrect because the relationship
number of days in a year divided by the accounts between the cost of capital and the prime rate may
receivable turnover (net credit sales/average not be relevant if the firm cannot borrow at the prime
receivables). The foregoing are some of the many rate.
ratios that can be used to evaluate working capital
management. Answer (D) is correct. Because these terms involve
an annual interest cost of over 36%, a company
Answer (C) is incorrect because times interest earned would not offer them unless it desperately needed
and the debt-to-equity ratio are measures of capital cash. Also, credit terms are typically somewhat
structure management. standardized within an industry. Thus, if most
companies in the industry offer similar terms, a firm
Answer (D) is incorrect because return on equity is a will likely be forced to match the competition or lose
measure of capital structure management. market share.
[46] Source: CMA 0691 1-6 [48] Source: CMA 0691 1-8
When a company offers credit terms of 2/10, net 30, the The result of the economic order quantity formula indicates
annual interest cost, based on a 360-day year, is the
Answer (A) is incorrect because the annual interest Answer (A) is incorrect because the annual quantity
cost is 36.7%. of inventory demanded is an input into the formula,
not the result.
Answer (B) is incorrect because the annual interest
cost is 36.7%. Answer (B) is incorrect because annual usage is a
determinant of annual demand, which is an input into
Answer (C) is incorrect because the annual interest the formula.
cost is 36.7%.
Answer (C) is incorrect because safety stock is not
Answer (D) is correct. Assume that the gross amount reflected in the basic EOQ formula.
of an invoice is $1,000. With a 2% discount, the
buyer will pay only $980 on the tenth day. Thus, the Answer (D) is correct. The EOQ model is a
seller is forgoing $20 to receive payment 20 days deterministic model that calculates the ideal order (or
sooner than would otherwise be required. The production lot) quantity given specified demand,
20-day interest rate is .0204 ($20/$980). The ordering or setup costs, and carrying costs. The
number of 20-day periods in a year is 18 (360/20). If model minimizes the sum of inventory carrying costs
and either ordering or production setup costs. investors. Commercial paper is more risky than
Treasury bills.
[49] Source: CMA 0691 1-9
The most direct way to prepare a cash budget for a [51] Source: CMA 0691 1-12
manufacturing firm is to include Which one of the following is not a characteristic of a
negotiable certificate of deposit? Negotiable certificates of
A. Projected sales, credit terms, and net income. deposit
B. Projected net income, depreciation, and goodwill A. Have a secondary market for investors.
amortization.
B. Are regulated by the Federal Reserve System.
C. Projected purchases, percentages of purchases
paid, and net income. C. Are usually sold in denominations of a minimum of
$100,000.
D. Projected sales and purchases, percentages of
collections, and terms of payments. D. Have yields considerably greater than bankers'
acceptances and commercial paper.
Answer (A) is incorrect because net income includes
noncash elements.
[52] Source: CMA 0692 1-22
Answer (B) is incorrect because net income includes The optimal level of inventory is affected by all of the
noncash elements, e.g., goodwill amortization and following except the
depreciation.
A. Usage rate of inventory per time period.
Answer (C) is incorrect because collection
percentages must be considered, and net income B. Cost per unit of inventory.
includes noncash elements.
C. Current level of inventory.
Answer (D) is correct. The most direct way of
preparing a cash budget requires incorporation of D. Cost of placing an order for merchandise.
sales projections and credit terms, collection
percentages, estimated purchases and payment Answer (A) is incorrect because the usage rate of
terms, and other cash receipts and disbursements. In inventory is a factor in determining how much
other words, preparation of the cash budget requires inventory to carry.
consideration of both inflows and outflows.
Answer (B) is incorrect because the cost of inventory
[50] Source: CMA 0691 1-10 affects carrying costs and a firm wants to minimize its
Commercial paper inventory carrying costs.
A. Has a maturity date greater than 1 year. Answer (C) is correct. The optimal level of inventory
is affected by the factors in the economic order
B. Is usually sold only through investment banking quantity (EOQ) model and delivery or production
dealers. lead times. These factors are the annual demand for
inventory, the carrying cost, which includes the
C. Ordinarily does not have an active secondary interest on funds invested in inventory, the usage rate,
market. and the cost of placing an order or making a
production run. The current level of inventory has
D. Has an interest rate lower than Treasury bills. nothing to do with the optimal inventory level.
Answer (A) is incorrect because commercial paper Answer (D) is incorrect because the cost of placing
usually has a maturity date of 270 days or less to an order affects how often orders are placed. A firm
avoid securities registration requirements. wants to minimize its ordering costs.
Answer (B) is incorrect because commercial paper is [53] Source: CMA 0692 1-23
often issued directly by the borrowing firm. If a firm's credit terms require payment within 45 days but
allow a discount of 2% if paid within 15 days (using a
Answer (C) is correct. Commercial paper is a form 360-day year), the approximate cost or benefit of the trade
of unsecured note that is sold by only the most credit terms is
creditworthy companies. It is issued at a discount
from its face value and has a maturity period of 270 A. 2%.
days or less. Commercial paper usually carries a low
interest rate in comparison to other means of B. 16%.
financing. SMA 4M, Understanding Financial
Instruments, observes that no general (active) C. 48%.
secondary market exists for commercial paper, but
that "most dealers or organizations will repurchase an D. 24%.
issue that they have sold."
Answer (A) is incorrect because the 2% savings is
Answer (D) is incorrect because interest rates must for 30 days only; the annualized interest rate paid for
be higher than those of Treasury bills to entice receiving the money early is about 24%.
refers to the difference between current assets and
Answer (B) is incorrect because 16% assumes eight current liabilities; fixed assets are not a component.
discount periods of 45 days each in a year rather than
12. Answer (C) is incorrect because total assets and total
liabilities are not components of working capital; only
Answer (C) is incorrect because 48% assumes current items are included.
15-day discount periods.
Answer (D) is incorrect because shareholders' equity
Answer (D) is correct. Assume that an invoice is due is not a component of working capital; only current
in 45 days. However, the seller allows a 2% cash items are included in the concept of working capital.
discount if the invoice is paid within 15 days. Given
early payment, the seller will receive their money at [56] Source: CMA 0692 1-26
least 30 days (45 - 15) sooner than the contract RLF Corporation had income before taxes of $60,000 for
requires. However, the seller is effectively paying 2% the year. Included in this amount were depreciation of
of the invoice price to receive the money 30 days $5,000, a charge of $6,000 for the amortization of bond
early. The approximate annual interest cost is 24% discounts, and $4,000 for interest expense. The estimated
because a 360-day year contains 12 periods of 30 cash flow for the period is
days each.
A. $60,000.
[54] Source: CMA 0692 1-24
The level of safety stock in inventory management depends B. $66,000.
on all of the following except the
C. $49,000.
A. Level of uncertainty of the sales forecast.
D. $71,000.
B. Level of customer dissatisfaction for back orders.
Answer (A) is incorrect because the cash flow for the
C. Cost of running out of inventory. period is greater than net income given noncash
expenses in the form of depreciation and bond
D. Cost to reorder stock. discount amortization.
Answer (A) is incorrect because the variability of Answer (B) is incorrect because $66,000 does not
sales during the lead time is a factor in the size of reflect the noncash expense for depreciation.
safety stocks.
Answer (C) is incorrect because the $5,000 of
Answer (B) is incorrect because the cost of depreciation and the $6,000 for amortization should
stockouts, including the opportunity cost of customer be added back to, not subtracted from, income.
dissatisfaction, is considered in determining safety
stock. Answer (D) is correct. To determine cash flow for
the period, all noncash expenses should be added
Answer (C) is incorrect because the cost of running back to net income. Adding the $5,000 of
out of inventory, often an opportunity cost, is a depreciation and the $6,000 of discount amortization
consideration. to the $60,000 of net income produces a cash flow
of $71,000.
Answer (D) is correct. The level of safety stock is
based on the delivery lead time and the variability of [57] Source: CMA 0692 1-27
sales during the lead time. The cost of stockouts, The following information applies to Brandon Company:
usually in the form of opportunity costs, is also
considered. Safety stocks are irrelevant to the Purchases Sales
economic order quantity. Thus, the cost to reorder, --------- --------
which is incorporated into the EOQ model, does not January $160,000 $100,000
affect the level of safety stock. February 160,000 200,000
March 160,000 240,000
[55] Source: CMA 0692 1-25 April 140,000 300,000
Net working capital is the difference between May 140,000 260,000
June 120,000 240,000
A. Current assets and current liabilities. A cash payment equal to 40% of purchases is made at the
time of purchase, and 30% is paid in each of the next 2
B. Fixed assets and fixed liabilities. months. Purchases for the previous November and
December were $150,000 per month. Payroll is 10% of
C. Total assets and total liabilities. sales in the month it occurs, and operating expenses are
20% of the following month's sales (July sales were
D. Shareholders' investment and cash. $220,000). Interest payments were $20,000 paid quarterly
in January and April. Brandon's cash disbursements for the
Answer (A) is correct. Net working capital is defined month of April were
as the difference between current assets and current
liabilities. Working capital is a measure of short-term A. $140,000.
solvency.
B. $152,000.
Answer (B) is incorrect because working capital
C. $200,000. Answer (D) is correct. Given sales of $27,000,000,
the average amount of daily sales must be $75,000
D. $254,000. ($27,000,000/360 days). The increased accounts
receivable balance is therefore $450,000 (6 days x
[58] Source: CMA 1292 1-19 $75,000). With an additional $450,000 of capital
Price Publishing is considering a change in its credit terms invested in receivables, the company's interest cost
from n/30 to 2/10, n/30. The company's budgeted sales for will increase by $36,000 per year (8% x $450,000).
the coming year are $24,000,000, of which 90% are Thus, the company must save at least $36,000 per
expected to be made on credit. If the new credit terms are year to justify the change in procedures.
adopted, Price estimates that discounts will be taken on
50% of the credit sales; however, uncollectible accounts [60] Source: CMA 1292 1-21
will be unchanged. The new credit terms will result in Dartmoor Company's budgeted sales for the coming year
expected discounts taken in the coming year of are $40,500,000, of which 80% are expected to be credit
sales at terms of n/30. Dartmoor estimates that a proposed
A. $216,000. relaxation of credit standards will increase credit sales by
20% and increase the average collection period from 30
B. $432,000. days to 40 days. Based on a 360-day year, the proposed
relaxation of credit standards will result in an expected
C. $240,000. increase in the average accounts receivable balance of
D. $480,000. A. $540,000.
Answer (C) is incorrect because only 90% of the Answer (B) is incorrect because $2,700,000 is the
sales are on credit; $240,000 would be correct only average amount in accounts receivable before any
if a cash discount were allowed on cash sales as well change in credit terms.
as credit sales.
Answer (C) is incorrect because $900,000 would
Answer (D) is incorrect because $480,000 is based have been the increase based solely on the increased
on the assumption that all sales will be discounted. number of days outstanding, assuming no increase in
sales.
[59] Source: CMA 1292 1-20
Best Computers believes that its collection costs could be Answer (D) is correct. Of the $40,500,000 of sales,
reduced through modification of collection procedures. This 80% are expected to be on credit, a total of
action is expected to result in a lengthening of the average $32,400,000. Average daily credit sales are
collection period from 28 days to 34 days; however, there therefore $90,000 ($32,400,000/360). If 30 days of
will be no change in uncollectible accounts. The company's sales are outstanding at any one time, the average
budgeted credit sales for the coming year are balance in accounts receivable is $2,700,000. If
$27,000,000, and short-term interest rates are expected to credit sales increase by 20%, the aforementioned
average 8%. To make the changes in collection procedures $90,000 average of daily sales will increase to
cost beneficial, the minimum savings in collection costs $108,000 (120% x $90,000). For an average
(using a 360-day year) for the coming year would have to collection period of 40 days, the average accounts
be receivable balance will be $4,320,000 (40 x
$108,000). Hence, the expected increase in the
A. $30,000. balance is $1,620,000 ($4,320,000 - $2,700,000).
Answer (A) is incorrect because a prepayment of Answer (A) is incorrect because factoring is the sale
expenses does not change current assets or current of receivables and therefore concerns cash inflows,
liabilities. Cash decreases by the same amount that not outflows.
prepaid rent increases.
Answer (B) is correct. A draft is a three-party
Answer (B) is correct. Working capital is the excess instrument in which one person (the drawer) orders a
of current assets over current liabilities. Refinancing a second person (the drawee) to pay money to a third
short-term debt with a long-term debt decreases person (the payee). A check is the most common
current liabilities, and the result is an increase in form of draft. It is an instrument payable on demand
working capital. in which the drawee is a bank. Consequently, a draft
can be used to delay the outflow of cash. A draft can
Answer (C) is incorrect because the purchase of be dated on the due date of an invoice and will not be
temporary investments does not affect total current processed by the drawee until that date, thereby
assets; cash is replaced by temporary investments, eliminating the necessity of writing a check earlier
another current asset. than the due date or using an EFT. Thus, the outflow
is delayed until the check clears the drawee bank.
Answer (D) is incorrect because the collection of a
receivable has no effect on total current assets. The
receivable is replaced by an equal amount of cash. [67] Source: CMA 1286 1-29
Finan Corporation's management is considering a plant
[65] Source: CMA 1293 1-20 expansion that will increase its sales and have
A lock-box system commensurate impact on its net working capital position.
The following information presents management's estimate
A. Reduces the need for compensating balances. of the impact the proposal will have on Finan.
Answer (A) is incorrect because the cost of not [70] Source: CIA 0593 IV-52
taking the cash discount is 8.7%. The company will Determining the amount and timing of conversions of
initially lose $2 by not taking the discount. This marketable securities to cash is a critical element of a
amount is partially offset by interest earned on $98 financial manager's performance. In terms of the rate of
for 50 days of $.817. Thus, the net cost is $1.183 return forgone on converted securities and the cost of such
($2.00 - $.817). Because a 360-day year has 7.2 transactions, the optimal amount of cash to be raised by
periods of 50 days each, the total annualized cost is selling securities is
$8.52 (7.2 x $1.183). The loss rate is about 8.7%
($8.52/$98). A. Inversely related to the rate of return forgone and
directly related to the cost of the transaction.
Answer (B) is correct. The company will initially lose
$2 by not taking the discount. This amount is partially B. Directly related to the rate of return forgone and
offset by interest earned on $98 for 50 days of directly related to the cost of the transaction.
$.817. Thus, the net cost is $1.183 ($2.00 - $.817).
Since a 360-day year has 7.2 fifty-day periods, the C. Directly related to the rate of return forgone and
total annualized cost is $8.52 (7.2 x $1.183). The inversely related to the cost of the transaction.
loss rate is about 8.7% ($8.52/$98).
D. Inversely related to the rate of return forgone and
Answer (C) is incorrect because the cost of not inversely related to the cost of the transaction.
taking the cash discount is 8.7%. The company will
initially lose $2 by not taking the discount. This Answer (A) is correct. The optimal amount of cash to
amount is partially offset by interest earned on $98 be raised by selling securities is calculated by a
for 50 days of $.817. Thus, the net cost is $1.183 formula similar to that used to determine the
($2.00 - $.817). Because a 360-day year has 7.2 economic order quantity for inventory.
periods of 50 days each, the total annualized cost is
$8.52 (7.2 x $1.183). The loss rate is about 8.7% + ス
($8.52/$98). C = (2(F)(T)/k)
+
Answer (D) is incorrect because the cost of not If: C = Cash to be raised
taking the cash discount is 8.7%. The company will T = Total cash needed for the period
initially lose $2 by not taking the discount. This F = Cost of making a securities trade
amount is partially offset by interest earned on $98 k = Opportunity cost of holding cash
for 50 days of $.817. Thus, the net cost is $1.183 The optimal amount of cash to be raised by selling
($2.00 - $.817). Because a 360-day year has 7.2 securities is inversely related to the rate of return
periods of 50 days each, the total annualized cost is forgone (opportunity cost) and directly related to the
$8.52 (7.2 x $1.183). The loss rate is about 8.7% cost of the transaction.
($8.52/$98).
Answer (B) is incorrect because a high (low)
opportunity cost results in a lower (higher) optimal
[69] Source: CIA 1192 IV-46 cash balance, whereas high (low) transaction costs
All decisions by financial managers should be driven by the result in a higher (lower) optimal cash balance.
primary goal to
Answer (C) is incorrect because a high (low)
A. Maximize revenues. opportunity cost results in a lower (higher) optimal
cash balance, whereas high (low) transaction costs
B. Minimize fixed costs and variable costs. result in a higher (lower) optimal cash balance.
Answer (C) is incorrect because stabilizing growth A. Increased to accommodate higher sales levels.
will not necessarily maximize stockholder wealth.
B. Reduced to offset the increased cost of carrying
Answer (D) is correct. The objective of the firm is to accounts receivable.
take those actions within legal and ethical limits that
will maximize the stockholders' wealth, i.e., the price C. Unaffected if safety stock is part of the current
per share of common stock. The market price of the quarterly order.
Effective rate = [1 + (.20/12)] - 1.0 = 21.94%.
D. Unaffected if the JIT inventory control system is
used. Answer (D) is incorrect because the note has an
effective rate, including compounding effects, of
Answer (A) is correct. Relaxing the credit policy for 21.94%. The following is the calculation:
customers will lead to increased sales because more
people will be eligible for more credit. As sales 12
increase, purchase orders will increase to Effective rate = [1 + (.20/12)] - 1.0 = 21.94%.
accommodate the higher sales levels.
[73] Source: CIA 1191 IV-56
Answer (B) is incorrect because inventory should be An example of secured short-term financing is
increased to accommodate higher sales levels.
A. Commercial paper.
Answer (C) is incorrect because safety stock is
based on expected sales, which are expected to rise. B. A warehouse receipt.
[72] Source: CIA 1188 IV-54 Answer (A) is incorrect because commercial paper is
A corporation is currently experiencing cash-flow problems a type of unsecured, short-term promissory note
and has determined that it is in need of short-term credit. It issued by large firms to other firms, insurance
companies, mutual funds, etc.
can either use its trade credit on $100,000 of accounts
payable with terms of 1/10, net 30 or a 30-day note with a Answer (B) is correct. A document of title is usually
20% annual simple interest rate. Which is the best issued by a bailee covering goods in the bailee's
alternative, and what is its effective rate of interest (rounded possession or care (UCC 1-201). It represents
to a whole percentage and using a 360-day year)? ownership of the goods and is ordinarily needed to
obtain the goods from the bailee. The two major
A. The trade credit. Its effective rate is 10%. types of documents of title are bills of lading (issued
by carriers) and warehouse receipts. A warehouse
B. The trade credit. Its effective rate is 20%. receipt is issued by a person engaged in the business
of storing goods for hire. Security for short-term
C. The note. Its effective rate is 17%. inventory financing can be arranged if the debtor
places its inventory under the control of the lender or
D. The note. Its effective rate is 20%. its agent (e.g., a public warehouse), and the lender
holds the warehouse receipts.
Answer (A) is incorrect because the effective trade
credit rate is 19.83% depending on the method of Answer (C) is incorrect because a revolving credit
calculation. agreement is a formal line of credit, usually with a
bank, that large firms often use.
Answer (B) is correct. The corporation can obtain
trade credit for 20 additional days by not paying Answer (D) is incorrect because a line of credit is an
within the discount period. Instead of paying $99,000 arrangement, which may be formal or informal,
to satisfy its obligation within 10 days, it can pay between a commercial bank and its customer
$100,000 at the end of 30 days. The corporation will concerning the maximum loan amount available.
thus incur $1,000 in interest to hold the $99,000 for
the 20 days. Because a 360-day year has 18 such [74] Source: CIA 0594 IV-51
periods, the interest rate is approximately 18.18% A manufacturing firm wants to obtain a short term loan and
[($1,000/$99,000) x 18]. However, if compounding has approached several lending institutions. All of the
effects are considered, the rate is higher. The potential lenders are offering the same nominal interest rate
effective rate, taking compounding into consideration, but the terms of the loans vary. Which of the following
is found using the following formula: combinations of loan terms will be most attractive for the
borrowing firm?
18
Effective rate = [1 + (1,000/99,000)] - 1.0 = 19.83%. A. Simple interest, no compensating balance.
In comparison, the 30-day note has an effective
annual rate of 21.94%, calculated as follows: B. Discount interest, no compensating balance.
Answer (D) is incorrect because the discount is 3%, Answer (D) is correct. A company will hold cash and
not 9%. marketable securities to facilitate business
transactions; cash is a medium of exchange. Cash and
[78] Source: CMA 0694 1-21 near-cash items are also held to meet future needs, to
A company obtained a short-term bank loan of $500,000 satisfy compensating balance requirements imposed
at an annual interest rate of 8%. As a condition of the loan, by lenders, and to provide a precautionary balance
the company is required to maintain a compensating for security purposes. Cash is usually not held in an
balance of $100,000 in its checking account. The checking attempt to earn maximum returns on investment
account earns interest at an annual rate of 3%. Ordinarily, because cash and marketable securities are not
the company maintains a balance of $50,000 in its account usually the highest paying investments.
for transaction purposes. What is the effective interest rate
of the loan? [80] Source: CMA 0694 1-23
An automated clearinghouse (ACH) electronic transfer is
A. 7.77% a(n)
Answer (A) is incorrect because the effective interest C. Computer-generated deposit ticket verifying
rate must exceed the 8% contract rate because not all deposit of funds.
of the borrowed funds are available for the debtor's
use. D. Check-like instrument drawn against the payor
and not against the bank.
Answer (B) is incorrect because 8.22% assumes
incremental earnings on the checking account of Answer (A) is correct. An ACH electronic funds
$3,000. transfer (EFT) is an electronic payment to a
company's account at a concentration bank. A
Answer (C) is incorrect because 9.25% is based on concentration bank is a large bank to which a
the assumption that the company ordinarily maintains company transfers funds from local depository banks.
a zero balance. These local banks operate the company's lockboxes
and thus serve as collection points. The transfer of
Answer (D) is correct. Of the $500,000 borrowed, funds to the concentration bank allows the company
the debtor has the use of only $450,000. The to take advantage of economies of scale in cash
compensating balance provision requires a minimum management. The use of ACHs facilitates
balance that is $50,000 greater than the balance that concentration banking. ACHs are electronic
the company usually maintains. At 8% on a $500,000 networks operated by the Federal Reserve (except
loan, the annual interest expense is $40,000. for the New York regional ACH association) that
However, this amount is reduced by the interest guarantee 1-day clearing.
earned on the extra $50,000 in the checking account.
At 3%, the extra $50,000 earns $1,500 per year. Answer (B) is incorrect because a check is not
Thus, the net expense is $38,500. The effective involved in an EFT.
interest rate is 8.555% ($38,500/$450,000).
Answer (C) is incorrect because an ACH transfer
[79] Source: CMA 0694 1-22 involves the actual transfer of funds electronically; it is
All of the following are valid reasons for a business to hold not just a computer-generated document.
cash and marketable securities except to
Answer (D) is incorrect because an EFT is not a
A. Satisfy compensating balance requirements. check-like instrument.
B. Maintain adequate cash needed for transactions. [81] Source: CMA 0694 1-24
Assume that each day a company writes and receives
C. Meet future needs. checks totaling $10,000. If it takes 5 days for the checks
to clear and be deducted from the company's account, and Assuming a 360-day year, the current price of a $100 U.S.
only 4 days for the deposits to clear, what is the float? Treasury bill due in 180 days on a 6% discount basis is
A. $10,000 A. $97.00
B. $0 B. $94.00
C. $(10,000) C. $100.00
D. $50,000 D. $93.00
Answer (A) is correct. The float period is the time Answer (A) is correct. The 6% discount rate is
between when a check is written and when it clears multiplied times the face amount of the Treasury bill
the payor's checking account. Check float results in to determine the amount of interest the lender will
an interest-free loan to the payor because of the earn. The interest on this Treasury bill is $3 (6% x .5
delay between payment by check and its deduction year x $100), thus the purchase price is $97 ($100 -
from the bank account. If checks written require one $3).
more day to clear than checks received, the net float
equals one day's receipts. The company will have Answer (B) is incorrect because the interest is for
free use of the money for one day. In this case, the 180 days, not a full year.
amount is $10,000.
Answer (C) is incorrect because the purchase price
Answer (B) is incorrect because the company enjoys will always be less than the face value when the
one day's net float because its checks clear more Treasury bill is sold at a discount.
slowly than its deposits.
Answer (D) is incorrect because the interest rate is
Answer (C) is incorrect because the net float is 6% per year.
positive. The company can write checks (up to
$10,000) even when it has no money because the [84] Source: CMA 0694 1-27
checks do not clear until a day after deposits clear. All of the following are inventory carrying costs except
Answer (A) is incorrect because U.S. Treasury bills Answer (C) is incorrect because opportunity cost of
are short-term marketable securities. inventory investment, and obsolescence and spoilage
are inventory holding costs.
Answer (B) is incorrect because Eurodollars are
short-term marketable securities. Answer (D) is correct. Inventory carrying costs are
incurred to hold inventory. Examples include the
Answer (C) is incorrect because commercial paper is costs of storage, insurance, security, inventory taxes,
a short-term marketable security. depreciation or rent of warehouse facilities,
obsolescence and spoilage, and the opportunity cost
Answer (D) is correct. Marketable securities are of inventory investment. Inspection costs are not
near-cash items used primarily for short-term related to the length of time inventory is held. They
investment. Examples include U. S. Treasury bills, are costs of taking delivery and are best classified as
Eurodollars, commercial paper, money-market ordering costs.
mutual funds with portfolios of short-term securities,
bankers' acceptances, floating rate preferred stock, [85] Source: CMA 0694 1-29
and negotiable CDs of U.S. banks. A convertible A firm that often factors its accounts receivable has an
bond is not a short-term investment because its agreement with its finance company that requires the firm to
maturity date is usually more than one year in the maintain a 6% reserve and charges 1% commission on the
future and its price can be influenced substantially by amount of receivables. The net proceeds would be further
changes in interest rates or by changes in the reduced by an annual interest charge of 10% on the monies
investee's stock price. advanced. Assuming a 360-day year, what amount of cash
(rounded to the nearest dollar) will the firm receive from the
[83] Source: CMA 0694 1-26 finance company at the time a $100,000 account that is
due in 90 days is turned over to the finance company?
Answer (B) is incorrect because buying a new plant
A. $93,000 with a 20-year mortgage has no effect on current
assets or current liabilities.
B. $90,000
Answer (C) is incorrect because cash collection of an
C. $83,700 account receivable increases one current asset and
decrease another by the same amount.
D. Refinancing a short-term note payable with a C. Any amount greater than $173.
two-year note payable.
D. Any amount greater than $62,500.
Answer (A) is incorrect because a cash payment of
payroll taxes decreases current assets and current Answer (A) is incorrect because the $25 transfer fee
liabilities by equal amounts. is covered by the interest on $62,500 for 2 days.
0 200 12%
Answer (B) is incorrect because $125,000 is The total cost of safety stock on an annual basis with a
required if collections are accelerated by only one safety stock level of 100 units is
day.
A. $1,750
Answer (C) is incorrect because the interest on $173
for 2 days is less than $.07. B. $1,950
Answer (A) is incorrect because 10% is the nominal C. Increases and the current ratio decreases.
rate.
D. Increases and the current ratio increases.
Answer (B) is correct. At 10%, the interest on a
$500,000 loan is $50,000 per year. However, the Answer (A) is incorrect because both working capital
$500,000 loan is effectively reduced to $450,000 of and the current ratio increase.
usable funds by the compensating balance
requirement. Thus, the borrower pays $50,000 of Answer (B) is incorrect because both working capital
interest for a $450,000 loan, an effective rate of and the current ratio increase.
11.1% ($50,000/$450,000).
Answer (C) is incorrect because both working capital
Answer (C) is incorrect because 9.1% equals and the current ratio increase.
$50,000 divided by $550,000.
Answer (D) is correct. Working capital is the excess
Answer (D) is incorrect because 12.2% equals of current assets over current liabilities. The current
$55,000 divided by $450,000. ratio equals current assets divided by current
liabilities. Selling stock for cash increases current
[95] Source: CMA 1294 1-24 assets and stockholders' equity, with no effect on
A company plans to tighten its credit policy. The new current liabilities. The result is an increase in working
policy will decrease the average number of days in capital and the current ratio.
collection from 75 to 50 days and will reduce the ratio of
credit sales to total revenue from 70% to 60%. The [97] Source: CMA 1295 1-1
company estimates that projected sales will be 5% less if The Stewart Co. uses the economic order quantity (EOQ)
the proposed new credit policy is implemented. If model for inventory management. A decrease in which one
projected sales for the coming year are $50 million, of the following variables would increase the EOQ?
calculate the dollar impact on accounts receivable of this
proposed change in credit policy. Assume a 360-day year. A. Annual sales.
Answer (C) is incorrect because a decrease in safety Answer (A) is incorrect because $6 million is the
stock levels will not affect the EOQ, although it might increase in cash, not the interest earned on that
lead to a different ordering point. additional cash.
Answer (D) is correct. The EOQ model minimizes Answer (B) is incorrect because $3 million is the
the total of ordering and carrying costs. The EOQ is amount of daily payments, not the savings.
calculated as follows:
Answer (C) is incorrect because the daily payments
1/2 should be multiplied by two, not divided by two.
レ ソ
ウ 2(Demand)(Order costs) ウ Answer (D) is correct. If cash outflows are $3 million
ウ ----------------------- ウ per day, holding cash 2 extra days means that
ウ Carrying costs per unit ウ average balances should increase by $6 million. At a
タ ル 10% interest rate, the additional $6 million would
generate interest revenue of $600,000 per year.
Increases in the numerator (demand or ordering Thus, if the system can be acquired for $600,000 or
costs) will increase the EOQ, whereas decreases in less, it would be beneficial to do so.
demand or ordering costs will decrease the EOQ.
Similarly, a decrease in the denominator (carrying [100] Source: CMA 1295 1-4
costs) will increase the EOQ. The average collection period for a firm measures the
number of days
[98] Source: CMA 1295 1-2
The working capital financing policy that subjects the firm A. After a typical credit sale is made until the firm
to the greatest risk of being unable to meet the firm's receives the payment.
maturing obligations is the policy that finances
B. For a typical check to "clear" through the banking
A. Fluctuating current assets with long-term debt. system.
B. Permanent current assets with long-term debt. C. Beyond the end of the credit period before a
typical customer payment is received.
C. Permanent current assets with short-term debt.
D. Before a typical account becomes delinquent.
D. Fluctuating current assets with short-term debt.
Answer (A) is correct. The average collection period
Answer (A) is incorrect because it is not particularly measures the number of days between the date of
risky to finance working capital needs from long-term sale and the date of collection. It should be related to
debt sources. a firm's credit terms. For example, a firm that allows
terms of 2/15, net 30, should have an average
Answer (B) is incorrect because it is not particularly collection period of somewhere between 15 and 30
risky to finance working capital needs from long-term days.
debt sources.
Answer (B) is incorrect because it describes the
Answer (C) is correct. Fluctuating current assets can concept of float.
often be financed with short-term debt because the
periodic liquidation of the assets provides funds to Answer (C) is incorrect because the average
pay off the debt. However, financing permanent collection period includes the total time before a
current assets with short-term debt is a risky strategy payment is received, including the periods both
because the assets may not be liquidated in time to before and after the end of the normal credit period.
pay off the debt at maturity.
Answer (D) is incorrect because it describes the
Answer (D) is incorrect because financing fluctuating normal credit period.
current assets with short-term debt is not as risky as
financing permanent current assets with short-term [101] Source: CMA 1295 1-6
debt. Jackson Distributors sells to retail stores on credit terms of
2/10, net 30. Daily sales average 150 units at a price of
[99] Source: CMA 1295 1-3 $300 each. Assuming that all sales are on credit and 60%
Average daily cash outflows are $3 million for Evans Inc. A of customers take the discount and pay on day 10 while the
new cash management system can add 2 days to the rest of the customers pay on day 30, the amount of
disbursement schedule. Assuming Evans earns 10% on Jackson's accounts receivable is
excess funds, how much should the firm be willing to pay
per year for this cash management system? A. $1,350,000
B. $990,000 B. With trade terms of 2/15, net 60, if the discount is
not taken, the buyer receives 45 days of free credit.
C. $900,000
C. The cost of not taking the discount is higher for
D. $810,000 terms of 2/10, net 60 than for 2/10, net 30.
Answer (A) is incorrect because 60% of the sales D. The cost of not taking a cash discount is generally
will be paid for within the 10-day discount period. higher than the cost of a bank loan.
Answer (B) is incorrect because $990,000 is based Answer (A) is incorrect because the cost of not
on a sales total of $1,650,000 for 30 days rather than taking a discount when terms are 2/10, net 30
$1,350,000. exceeds 36% annually, which is higher than the prime
rate has ever been.
Answer (C) is incorrect because $900,000 is based
on a sales total of $1,650,000 for 30 days rather than Answer (B) is incorrect because the buyer is paying
$1,350,000. the amount of discount not taken in exchange for the
extra 45 days of credit.
Answer (D) is correct. The firm has daily sales of
$45,000 consisting of 150 units at $300 each. For 30 Answer (C) is incorrect because paying 2% for 20
days, sales total $1,350,000. Forty percent of these days of credit is more expensive than paying 2% for
sales, or $540,000, will be uncollected because 50 days of the same amount of credit.
customers do not take their discounts. The remaining
60%, or $810,000, will be paid within the discount Answer (D) is correct. Payments should be made
period. However, by the end of 30 days, only 2/3 of within the discount periods if the cost of not taking
the $810,000 will be collected because the sales discounts exceeds the firm's cost of capital. For
from days 21 through 30 are still within the discount example, failing to take a discount when terms are
period. Therefore, an additional $270,000 2/10, net 30 means that the firm is paying an effective
($810,000 - $540,000) will still be uncollected after annual interest rate exceeding 36%. Thus, the cost of
the 30th day, but will be subject to a discount. In not taking a discount is usually higher than the cost of
total, the average receivable balance is $810,000, a bank loan.
consisting of $540,000 on which no discount will be
taken and $270,000 that will be paid within the [104] Source: CMA 1295 1-8
discount period. Shown below is a forecast of sales for Cooper Inc. for the
first 4 months of the year (all amounts are in thousands of
[102] Source: CMA 1295 1-5 dollars).
When the Economic Order Quantity (EOQ) model is used
for a firm that manufactures its inventory, ordering costs January February March April
consist primarily of ------- -------- ----- -----
Cash sales $15 $24 $18 $14
A. Insurance and taxes. Sales on credit 100 120 90 70
On average, 50% of credit sales are paid for in the month
B. Obsolescence and deterioration. of sale, 30% in the month following the sale, and the
remainder is paid 2 months after the month of sale.
C. Storage and handling. Assuming there are no bad debts, the expected cash inflow
for Cooper in March is
D. Production set-up.
A. $138,000
Answer (A) is incorrect because insurance and taxes
are carrying costs. B. $122,000
[103] Source: CMA 1295 1-7 Answer (C) is correct. Cash inflows for March
Which one of the following statements concerning cash would consist of 50% of March credit sales (50% x
discounts is correct? $90 = $45), plus 30% of February credit sales (30%
x $120 = $36), plus 20% of January credit sales
A. The cost of not taking a 2/10, net 30 cash (20% x $100 = $20), plus cash sales for March of
discount is usually less than the prime rate. $18. Consequently, total collections equal $119,000.
addition, Dixon is required to maintain a 20%
Answer (D) is incorrect because $108,000 is the compensating balance in its checking account. Assuming
total sales for March, not the total cash collections for the company would normally maintain a zero balance in its
March. checking account, the effective interest rate on the loan is
Answer (A) is incorrect because 250 is the maximum Answer (C) is incorrect because 20% is the
inventory level. percentage of the required compensating balance.
Answer (B) is correct. If safety stock is 50 units, the Answer (D) is correct. The requirement to maintain a
receipt of an order should increase the inventory to compensating balance of 20% of the $300,000 loan
250. That amount will decline to 50 just prior to the means that the borrower has effective use of only
receipt of the next order. Thus, the average inventory 80% of the loan, or $240,000. The 8% interest rate
would be the average of 250 and 50 [(250 + 50)/2], applied to a $300,000 loan requires an annual
or 150 units. interest expenditure of $24,000. In turn, paying
$24,000 for the use of $240,000 indicates an
Answer (C) is incorrect because 125 units assumes effective interest rate of 10%.
an EOQ of 250 units and no safety stock.
[108] Source: CMA 1295 1-11
Answer (D) is incorrect because 100 units assumes Elan Corporation is considering borrowing $100,000 from
no safety stock. a bank for 1 year at a stated interest rate of 9%. What is
the effective interest rate to Elan if this borrowing is in the
[106] Source: CMA 1295 1-9 form of a discounted note?
Which one of the following financial instruments generally
provides the largest source of short-term credit for small A. 8.10%
firms?
B. 9.00%
A. Installment loans.
C. 9.81%
B. Commercial paper.
D. 9.89%
C. Trade credit.
Answer (A) is incorrect because the lesser amount of
D. Bankers' acceptances. funds available on a discounted note means the
effective rate will be higher than the contract rate.
Answer (A) is incorrect because installment loans are
usually a longer-term source of financing and are Answer (B) is incorrect because 9% is the nominal
more difficult to acquire than trade credit. rate (discount rate).
Answer (B) is incorrect because commercial paper is Answer (C) is incorrect because 9.81% equals the
normally used only by large companies with high nominal rate multiplied by 9%.
credit ratings.
Answer (D) is correct. Applying the 9% interest rate
Answer (C) is correct. Trade credit is a spontaneous to a $100,000 loan results in interest expense of
source of financing because it arises automatically as $9,000. If the loan is processed in the form of a
part of a purchase transaction. Because of its ease in discounted note, the interest will be deducted from
use, trade credit is the largest source of short-term the proceeds of the loan. Thus, the $9,000 of interest
financing for many firms both large and small. will be deducted from the $100,000 note, resulting in
loan proceeds of $91,000. The borrower is paying
Answer (D) is incorrect because bankers' $9,000 for a loan of $91,000, resulting in an effective
acceptances are drafts drawn on bank deposits; the interest rate of 9.89%.
acceptance is a guarantee of payment at maturity.
[109] Source: CMA 1295 1-12
[107] Source: CMA 1295 1-10 When managing cash and short-term investments, a
The Dixon Corporation has an outstanding 1-year bank corporate treasurer is primarily concerned with
loan of $300,000 at a stated interest rate of 8%. In
A. Maximizing rate of return.
A. Accounts payable.
B. Minimizing taxes.
B. Mortgage bonds.
C. Investing in Treasury bonds since they have no
default risk. C. Accounts receivable.
Answer (A) is incorrect because most companies are Answer (A) is correct. Trade credit is a spontaneous
not in business to earn high returns on liquid assets source of financing because it arises automatically as
(i.e., they are held to facilitate operations). part of a purchase transaction. Because of its ease in
use, trade credit is the largest source of short-term
Answer (B) is incorrect because the holding of cash financing for many firms both large and small.
and cash-like assets is not a major factor in
controlling taxes. Answer (B) is incorrect because mortgage bonds and
debentures do not arise automatically as a result of a
Answer (C) is incorrect because investments in purchase transaction.
Treasury bonds do not have sufficient liquidity to
serve as short-term assets. Answer (C) is incorrect because the use of
receivables as a financing source requires an
Answer (D) is correct. Cash and short-term extensive factoring arrangement and often involves
investments are crucial to a firm's continuing success. the creditor's evaluation of the credit ratings of the
Sufficient liquidity must be available to meet payments borrower's customers.
as they come due. At the same time, liquid assets are
subject to significant control risk. Therefore, liquidity Answer (D) is incorrect because mortgage bonds and
and safety are the primary concerns of the treasurer debentures do not arise automatically as a result of a
when dealing with highly liquid assets. Cash and purchase transaction.
short-term investments are held because of their
ability to facilitate routine operations of the company.
These assets are not held for purposes of achieving [112] Source: CMA 0696 1-10
investment returns. A company uses the following formula in determining its
optimal level of cash.
[110] Source: CMA 1295 1-14
Foster Inc. is considering implementing a lock-box * 2bT
collection system at a cost of $80,000 per year. Annual C = square root of -----
sales are $90 million, and the lockbox system will reduce i
collection time by 3 days. If Foster can invest funds at 8%, If: b = fixed cost per transaction
should it use the lockbox system? Assume a 360-day year. i = interest rate on marketable securities
T = total demand for cash over a period of time
A. Yes, producing savings of $140,000 per year. This formula is a modification of the economic order
quantity (EOQ) formula used for inventory management.
B. Yes, producing savings of $60,000 per year. Assume that the fixed cost of selling marketable securities is
$10 per transaction and the interest rate on marketable
C. No, producing a loss of $20,000 per year. securities is 6% per year. The company estimates that it will
make cash payments of $12,000 over a one-month period.
D. No, producing a loss of $60,000 per year. What is the average cash balance (rounded to the nearest
dollar)?
Answer (A) is incorrect because $140,000 equals
the $80,000 cost plus the investment earnings. A. $1,000
Answer (A) is incorrect because lower rates are an C. Offsetting the benefit of current assets and current
advantage of commercial paper. liabilities against the probability of technical
insolvency.
Answer (B) is incorrect because avoidance of
compensating balance requirements is an advantage D. Maintaining a high proportion of liquid assets to
of commercial paper. total assets in order to maximize the return on total
investments.
Answer (C) is correct. Commercial paper is a
short-term, unsecured note payable issued in large Answer (A) is incorrect because capital structure and
denominations by major companies with excellent dividends relate to capital structure finance, not
credit ratings. Maturities usually do not exceed 270 working capital finance.
days. Commercial paper is a lower cost source of
funds than bank loans, and no compensating balances Answer (B) is incorrect because short-term debt is
are required. Commercial paper provides a broad usually less expensive than long-term debt.
and efficient distribution of debt, and costly financing
arrangements are avoided. The market is not open to Answer (C) is correct. Working capital finance
all companies because only major corporations with concerns the determination of the optimal level, mix,
high credit ratings can participate. and use of current assets and current liabilities. The
objective is to minimize the cost of maintaining
Answer (D) is incorrect because broad debt liquidity, while guarding against the possibility of
distribution is an advantage of commercial paper. technical insolvency. Technical insolvency is defined
as the inability to pay debts as they come due.
[117] Source: CMA 0696 1-15 Answer (D) is incorrect because liquid assets do not
Spotech Co.'s budgeted sales and budgeted cost of sales ordinarily earn high returns relative to long-term
for the coming year are $212,000,000 and $132,500,000, assets, so holding the former will not maximize the
respectively. Short-term interest rates are expected to return on total assets.
average 5%. If Spotech could increase inventory turnover
from its current 8.0 times per year to 10.0 times per year, [119] Source: CMA 0696 1-17
its expected cost savings in the current year would be The amount of inventory that a company would tend to
hold in stock would increase as the
A. $165,625
Answer (A) is correct. If cost of sales is D. Cost of running out of stock decreases.
$132,500,000, and the inventory turnover rate is 8.0
times per year, the average inventory is $16,562,500 Answer (A) is incorrect because permanently lower
($132,500,000/8). If the turnover increases to 10.0 sales, more predictable sales, decreased stockout
times annually, the average inventory will decline to costs, and decreased inventory transit time are
$13,250,000 ($132,500,000/10), a decrease of reasons for decreasing inventory.
$3,312,500. At a 5% rate, reducing working capital
by $3,312,500 will save the company $165,625 (.05 Answer (B) is correct. Inventory management
x $3,312,500). attempts to minimize the total costs of ordering,
carrying inventory, and stockouts. Thus, a firm incurs
Answer (B) is incorrect because the faster turnover carrying costs to reduce ordering and stockout costs.
reduces working capital and releases funds for other If the cost of carrying inventory declines, the
uses. inventory level must increase to minimize total
inventory costs.
Answer (C) is incorrect because $3,312,500 is the
decrease in average inventory. Answer (C) is incorrect because permanently lower
sales, more predictable sales, decreased stockout
Answer (D) is incorrect because $828,125 is 5% of costs, and decreased inventory transit time are
$16,562,500. reasons for decreasing inventory.
[118] Source: CMA 0696 1-16 Answer (D) is incorrect because permanently lower
Determining the appropriate level of working capital for a sales, more predictable sales, decreased stockout
firm requires costs, and decreased inventory transit time are
reasons for decreasing inventory.
fact that the 2% fee recurs every month.
[120] Source: CMA 0696 1-29
All of the following statements in regard to working capital Answer (C) is incorrect because 14% miscalculates
are correct except the factor fee and the savings from reduced collection
costs.
A. Current liabilities are an important source of
financing for many small firms. Answer (D) is correct. In an average month, the
company will receive $80,000 at the time the
B. Profitability varies inversely with liquidity. receivables are sent to the factor. Over a year's time,
the interest on this average advance of $80,000
C. The hedging approach to financing involves would be $8,000 at 10% interest. In addition, the
matching maturities of debt with specific financing factor will charge a 2% factor fee, or $24,000
needs. ($100,000 x 12 x .02) over the course of a year.
However, this $24,000 fee is offset by the $18,000
D. Financing permanent inventory buildup with savings in collection expenses, producing a net outlay
long-term debt is an example of an aggressive of only $6,000. Adding the $6,000 to the $8,000 of
working capital policy. interest produces an annual net cost of $14,000.
Dividing the $14,000 cost by the $80,000 of
Answer (A) is incorrect because current liabilities, advanced funds results in a cost of 17.5%.
e.g., trade credit, is a major source of funds for small
firms. [122] Source: CMA 1296 1-5
A working capital technique that increases the payable float
Answer (B) is incorrect because liquid investments and therefore delays the outflow of cash is
tend to have low returns.
A. Concentration banking.
Answer (C) is incorrect because matching of asset
and liability maturities is a moderate policy that B. A draft.
minimizes risk. The expectation is that cash flows
from the assets will be available to meet obligations C. Electronic Data Interchange (EDI).
for the liabilities.
D. A lockbox system.
Answer (D) is correct. Financing permanent
inventory buildup, which is essentially a long-term Answer (A) is incorrect because concentration
investment, with long-term debt is a moderate or banking, a lockbox system, and the use of a local
conservative working capital policy. An aggressive post office box are techniques used to accelerate
policy uses short-term, relatively low-cost debt to cash receipts.
finance the inventory buildup. It focuses on high
profitability potential, despite high risk and low Answer (B) is correct. Payment by draft, a
liquidity. An aggressive policy reduces the current three-party instrument in which the drawer orders the
ratio and accepts a higher risk of short-term lack of drawee to pay money to the payee, is a means of
liquidity. Financing inventory with long-term debt slowing cash outflows. A check is the most common
increases the current ratio and accepts higher type of draft. Check float arises from the delay
borrowing costs in exchange for greater liquidity and between an expenditure and the clearing of the check
lower risk. through the banking system.
[121] Source: CMA 0696 1-30 Answer (C) is incorrect because EDI is the
A company enters into an agreement with a firm that will communication of electronic documents directly from
factor the company's accounts receivable. The factor a computer in one entity to a computer in another
agrees to buy the company's receivables, which average entity. Thus, EDI expedites cash payments. The
$100,000 per month and have an average collection period payee receives the money almost instantaneously.
of 30 days. The factor will advance up to 80% of the face
value of receivables at an annual rate of 10% and charge a Answer (D) is incorrect because concentration
fee of 2% on all receivables purchased. The controller of banking, a lockbox system, and the use of a local
the company estimates that the company would save post office box are techniques used to accelerate
$18,000 in collection expenses over the year. Fees and cash receipts.
interest are not deducted in advance. Assuming a 360-day
year, what is the annual cost of financing? [123] Source: CMA 1296 1-6
A change in credit policy has caused an increase in sales,
A. 10.0% an increase in discounts taken, a decrease in the amount of
bad debts, and a decrease in the investment in accounts
B. 12.0% receivable. Based upon this information, the company's
Answer (A) is incorrect because 10% overlooks the C. Accounts receivable turnover has decreased.
factor fee.
D. Working capital has increased.
Answer (B) is incorrect because 12% overlooks the
Answer (A) is correct. An increase in discounts taken CyberAge Outlet, a relatively new store, is a cafe that
accompanied by declines in receivables balances and offers customers the opportunity to browse the Internet or
doubtful accounts all indicate that collections on the play computer games at their tables while they drink coffee.
increased sales have been accelerated. Accordingly, The customer pays a fee based on the amount of time spent
the average collection period must have declined. The signed on to the computer. The store also sells books,
average collection period is a ratio calculated by tee-shirts, and computer accessories. CyberAge has been
dividing the number of days in a year (365) by the paying all of its bills on the last day of the payment period,
receivable turnover. Thus, the higher the turnover, the thus forfeiting all supplier discounts. Shown below are data
shorter the average collection period. The turnover on CyberAge's two major vendors, including average
increases when either sales (the numerator) increase, monthly purchases and credit terms.
or receivables (the denominator) decrease.
Accomplishing both higher sales and a lower Average
receivables increases the turnover and results in a Monthly
shorter collection period. Vendor Purchases Credit Terms
---------- --------- ------------
Answer (B) is incorrect because a decrease in the Web Master $25,000 2/10, net 30
percentage discount offered provides no incentive for Softidee 50,000 5/10, net 90
early payment.
[125] Source: CMA 1296 1-10
Answer (C) is incorrect because accounts receivable (Refers to Fact Pattern #2)
turnover (sales/average receivables) has increased. Assuming a 360-day year and that CyberAge continues
paying on the last day of the credit period, the company's
Answer (D) is incorrect because no information is weighted-average annual interest rate for trade credit
given relative to working capital elements other than (ignoring the effects of compounding) for these two
receivables. Both receivables and cash are elements vendors is
of working capital, so an acceleration of customer
payments will have no effect on working capital. A. 27.0%
B. Increase in the ratio of current liabilities to Answer (A) is incorrect because 27.0% is based on
noncurrent liabilities. weights of $25,000 and $50,000.
C. Increase in the ratio of current assets to units of Answer (B) is correct. If the company pays Web
output. Master within 10 days, it will save $500 (2% x
$25,000). Thus, the company is effectively paying
D. Increase in funds invested in common stock and a $500 to retain $24,500 ($25,000 - $500) for 20
decrease in funds invested in marketable securities. days (30 - 10). The annualized interest rate on this
borrowing is 36.7346% [($500/$24,500) x (360
Answer (A) is incorrect because a decrease in the days/20 days)]. Similarly, the company is, in effect,
acid-test ratio suggests an aggressive policy. A paying Softidee $2,500 (5% x $50,000) to hold
conservative company wants a higher acid-test ratio, $47,500 ($50,000 - $2,500) for 80 days (90 - 10).
that is, more liquid assets relative to liabilities. The annualized rate on this borrowing is 23.6842%
[($2,500/$47,500) x (360 days/80 days)]. The
Answer (B) is incorrect because a conservative average amount borrowed from Web Master is
company wants working capital to be financed from $16,333.33 [1 month x $24,500 x (20 days/30
long-term sources. days)], and the average amount borrowed from
Softidee is $126,666.67 [3 months x $47,500 x (80
Answer (C) is correct. A conservative working days/90 days)]. Thus, the weighted average of these
capital policy minimizes liquidity risk by increasing two rates based on average borrowings is 25.2%
working capital (current assets - current liabilities). {[36.7346% x $16,333.33) + (23,6842 x
The result is that the company forgoes the potentially $126,666.67)]/($16,333.33 + $126,666.67)}. This
higher returns available from using the additional calculation, however, understates the true cost of not
working capital to acquire long-term assets. A taking the discount because it does not consider the
conservative working capital policy is characterized effects of compounding.
by a higher current ratio (current assets/current
liabilities) and acid-test ratio (quick assets/current Answer (C) is incorrect because 28.0% is based on
liabilities). Thus, the company will increase current weights of $24,500 and $47,500.
assets or decrease current liabilities. A conservative
policy finances assets using long-term or permanent Answer (D) is incorrect because 30.2% is an
funds rather than short-term sources. unweighted average of the two interest rates.
Answer (D) is incorrect because a conservative [126] Source: CMA 1296 1-11
company seeks more liquid (marketable) investments. (Refers to Fact Pattern #2)
Should CyberAge use trade credit and continue paying at
[Fact Pattern #2] the end of the credit period?
credit depends on the credit terms and the price paid.
A. Yes, if the cost of alternative short-term financing A seller with generous payment terms may charge a
is less. higher price for its merchandise.
B. Yes, if the firm's weighted-average cost of capital [128] Source: CMA 1296 1-13
is equal to its weighted-average cost of trade credit. Which one of the following statements is most correct if a
seller extends credit to a purchaser for a period of time
C. No, if the cost of alternative long-term financing is longer than the purchaser's operating cycle? The seller
greater.
A. Will have a lower level of accounts receivable than
D. Yes, if the cost of alternative short-term financing those companies whose credit period is shorter than
is greater. the purchaser's operating cycle.
Answer (A) is incorrect because the company should B. Is, in effect, financing more than just the
continue the current practice unless alternative purchaser's inventory needs.
short-term financing is available at a lower rate.
C. Can be certain that the purchaser will be able to
Answer (B) is incorrect because the convert the inventory into cash before payment is
weighted-average cost of capital is usually a concern due.
in capital budgeting and is not as important in the
decision process as the marginal cost of capital. D. Has no need for a stated discount rate or credit
Furthermore, trade credit is just one element in the period.
firm's financing structure. An optimal mix of financing
sources may require that trade credit be obtained at Answer (A) is incorrect because a seller that extends
less than the weighted-average cost of capital. long-term credit will have a higher level of receivables
than a firm with a shorter credit period.
Answer (C) is incorrect because the company should
maintain its current practice if the cost of alternative Answer (B) is correct. The normal operating cycle is
long-term financing is higher. defined as the period from the acquisition of inventory
to the collection of the account receivable. If trade
Answer (D) is correct. The company is currently credit is for a period longer than the normal operating
paying an annual rate of 25.2% (see previous cycle, the seller must therefore be financing more than
question) to obtain trade credit and pay at the end of just the purchase of inventory.
the credit period. This policy should be continued if
trade credit is the only source of financing, or if other Answer (C) is incorrect because the seller is not
sources are available only at a higher rate. guaranteed that a purchaser will resell the
merchandise.
[127] Source: CMA 1296 1-12
Answer (D) is incorrect because offering a discount
Which one of the following statements about trade credit is may accelerate payment.
correct? Trade credit is
[Fact Pattern #3]
A. Not an important source of financing for small The Frame Supply Company has just acquired a large
firms. account and needs to increase its working capital by
$100,000. The controller of the company has identified the
B. A source of long-term financing to the seller. four sources of funds given below.
C. Subject to risk of buyer default. A. Pay a factor to buy the company's receivables,
which average $125,000 per month and have an average
D. Usually an inexpensive source of external collection period of 30 days. The factor will
financing. advance up to 80% of the face value of receivables
at 10% and charge a fee of 2% on all receivables
Answer (A) is incorrect because trade credit is an purchased. The controller estimates that the firm
important source of financing for small firms. would save $24,000 in collection expenses over the
year. Assume the fee and interest are not deductible
Answer (B) is incorrect because trade credit is in advance.
ordinarily a short-term source of financing. B. Borrow $110,000 from a bank at 12% interest. A 9%
compensating balance would be required.
Answer (C) is correct. Trade credit is a spontaneous C. Issue $110,000 of 6-month commercial paper to net
source of financing because it arises automatically as $100,000. (New paper would be issued every 6 months.)
part of a purchase transaction. The terms of payment D. Borrow $125,000 from a bank on a discount basis at
are set by the supplier, but trade credit usually 20%. No compensating balance would be required.
requires payment within a short period of time. Trade Assume a 360-day year in all of your calculations.
credit is an important source of credit for all
businesses but especially for buyers, such as small [129] Source: CMA 1296 1-14
businesses, that might not have access to other credit (Refers to Fact Pattern #3)
markets. Like all forms of financing, trade credit is The cost of Alternative A. is
subject to the risk of buyer default.
A. 10.0%
Answer (D) is incorrect because the cost of trade
B. 12.0% six months.
[130] Source: CMA 1296 1-15 Answer (A) is incorrect because the effective rate
(Refers to Fact Pattern #3) must exceed the contract rate of 20%.
The cost of Alternative B. is
Answer (B) is correct. The company will receive
A. 9.0% $100,000 (80% x $125,000) at an annual cost of
$25,000, so the annual interest rate is 25%
B. 12.0% ($25,000/$100,000).
Answer (C) is correct. Even though the company will I. Days' sales in receivables x accounts receivable
borrow $110,000, it will have use of only $100,100 turnover.
because a 9% compensating balance, or $9,900, II. Average daily sales x average collection period.
must be maintained at all times. Consequently, the III. Net sales/average gross receivables.
effective annual interest rate is 13.2% [(12% x A. I only.
$110,000)/$100,100].
B. I and II only.
Answer (D) is incorrect because 21% is the sum of
the contract rate and the compensating balance C. II only.
requirement.
D. II and III only.
[131] Source: CMA 1296 1-16
(Refers to Fact Pattern #3) Answer (A) is incorrect because Alternative I cannot
The cost of Alternative C. is be correct. Neither of the multiplicands is a dollar
figure, so the product could not be the dollar balance
A. 9.1% of receivables.
[134] Source: CMA 0697 1-7 [136] Source: CMA 0697 1-9
Which one of the following transactions would increase the Clauson Inc. grants credit terms of 1/15, net 30 and
current ratio and decrease net profit? projects gross sales for next year of $2,000,000. The
credit manager estimates that 40% of their customers pay
A. A federal income tax payment due from the on the discount date, 40% on the net due date, and 20%
previous year is paid. pay 15 days after the net due date. Assuming uniform sales
and a 360-day year, what is the projected days' sales
B. A stock dividend is declared. outstanding (rounded to the nearest whole day)?
Answer (A) is incorrect because the length of the Answer (A) is correct. The current ratio is calculated
extra credit period is 20 days, not 30 days. by dividing current assets by current liabilities. Thus,
any transaction that changes current assets or current
Answer (B) is incorrect because the length of the liabilities changes the current ratio. A cash advance to
extra credit period is 20 days, not 30 days. a divisional office does not change current assets
because the cash (a current asset) will be replaced by
Answer (C) is incorrect because 36.0% calculates a receivable (a current asset). Current liabilities are
the interest rate based on the full invoice price. also unaffected, so the cash advance affects neither
the current ratio nor total current assets.
Answer (D) is correct. On a $1,000 invoice, the
Answer (B) is incorrect because the declaration of a than the $125,000 flat fee in (B). Given that the
cash dividend creates a current liability, which annual collections equal $453,600,000 ($1,800 x
reduces the current ratio. 700 x 360), (C) is also less desirable because the
annual fee would be $136,080 (.03% x
Answer (C) is incorrect because retiring short-term $453,600,000). The best option is therefore to
notes with cash reduces current assets and current maintain a compensating balance of $1,750,000
liabilities. when the cost of funds is 7%, resulting in a total cost
of $122,500 (.07 x $1,750,000).
Answer (D) is incorrect because selling a fully
depreciated asset increases cash, a current asset. [140] Source: CMA 0697 1-14
The sales manager at Ryan Company feels confident that, if
[138] Source: CMA 0697 1-12 the credit policy at Ryan's were changed, sales would
Which one of the following would not be considered a increase and, consequently, the company would utilize
carrying cost associated with inventory? excess capacity. The two credit proposals being
considered are as follows:
A. Insurance costs.
A. A $0.50 fee per check. Answer (C) is incorrect because the impact on the
current customer base of extending terms to only
B. A flat fee of $125,000 per year. certain customers is relevant. The current customers
may demand the same terms.
C. A fee of 0.03% of the amount collected.
Answer (D) is incorrect because existing loan
D. A compensating balance of $1,750,000. agreements may require Ryan to maintain certain
ratios at stated levels. Thus, Ryan's ability to increase
Answer (A) is incorrect because the annual cost is receivables and possible bad debt losses may be
$126,000. limited.
Answer (B) is incorrect because the annual cost is [141] Source: CMA 0697 1-15
$125,000. The treasury analyst for Garth Manufacturing has estimated
the cash flows for the first half of next year (ignoring any
Answer (C) is incorrect because the annual cost is short-term borrowings) as follows.
$136,080.
Cash (millions)
Answer (D) is correct. Multiplying 700 checks times -------------------
360 days results in a total of 252,000 checks per Inflows Outflows
year. Accordingly, under (A), total annual cost is ------- --------
$126,000 ($.50 x 252,000), which is less desirable January $2 $1
February 2 4 Current portion, long-term notes payable 65,000
March 2 5 What is the maximum amount MFC can pay in cash
April 2 3 dividends per share and maintain a minimum current ratio of
May 4 2 2 to 1? Assume that all accounts other than cash remain
June 5 3 unchanged.
Garth has a line of credit of up to $4 million on which it
pays interest monthly at a rate of 1% of the amount utilized. A. $2.05
Garth is expected to have a cash balance of $2 million on
January 1 and no amount utilized on its line of credit. B. $2.50
Assuming all cash flows occur at the end of the month,
approximately how much will Garth pay in interest during C. $3.35
the first half of the year?
D. $3.80
A. Zero.
Answer (A) is incorrect because $2.05 fails to
B. $61,000 include prepaid assets (prepaid expenses) as current
assets.
C. $80,000
Answer (B) is correct. Before the dividend, total
D. $132,000 current assets equal $2,050,000 ($455,000 cash +
$900,000 receivables + $650,000 inventory +
$45,000 prepaid assets). Current liabilities total
Answer (A) is incorrect because interest must be $900,000 ($285,000 accrued liabilities + $550,000
paid monthly when the credit line is used in April, accounts payable + $65,000 current portion of
May, and June. long-term debt). The payment of the cash dividend
will not change current liabilities, so a current ratio of
Answer (B) is correct. The sum of the beginning 2 to 1 requires that current assets be maintained at a
balance and inflows exceeds the outflows for the first minimum of $1,800,000 (2 x $900,000). Thus, cash
2 months. At the end of March, however, Garth must can decrease by $250,000 ($2,050,000 -
use $2,000,000 of its line of credit ($2,000,000 $1,800,000). The maximum per-share rate is $2.50
beginning balance + $6,000,000 inflows - ($250,000/100,000 shares).
$10,000,000 outflows). Thus, interest for April is
$20,000 (1% x $2,000,000). The net cash outflow Answer (C) is incorrect because $3.35 fails to
for April (ignoring short-term borrowings) is include the current portion of long-term debt as a
$1,000,000 of an additional $1,000,000 of the line of current liability.
credit. However, the $20,000 of interest for April
must also be paid, so the amount of the line of credit Answer (D) is incorrect because $3.80 fails to
used in May is $3,020,000 ($2,000,000 + consider prepaid assets as a current asset and the
$1,000,000 + $20,000). Interest for May is current portion of long-term debt as a current liability.
therefore $30,200 (1% x $3,020,000). Given the net
cash inflow for May of $2,000,000 (again ignoring [143] Source: CMA 0697 1-19
short-term borrowings) and the borrowing of A company obtained a short-term bank loan of $250,000
$30,200 to pay the interest for May, the amount of at an annual interest rate of 6%. As a condition of the loan,
the line of credit used in June is $1,050,200. Interest the company is required to maintain a compensating
in June is $10,502 (1% x $1,050,200), and total balance of $50,000 in its checking account. The company's
interest is $60,702 ($20,000 + $30,200 + $10,502). checking account earns interest at an annual rate of 2%.
Consequently, the closest answer is $61,000. Ordinarily, the company maintains a balance of $25,000 in
its checking account for transaction purposes. What is the
Answer (C) is incorrect because the company would effective interest rate of the loan?
repay the credit line at the end of months with a
positive cash flow. A. 6.44%
Answer (B) is incorrect because the savings on the [146] Source: CMA 0689 1-25
premium account is $16. All of the following are examples of imputed costs except
Answer (C) is incorrect because the savings on the A. The stated interest paid on a bank loan.
premium account is $16.
B. The use of the firm's internal cash funds to
Answer (D) is correct. The standard account will purchase assets.
cost $10 per month plus $8 in check charges ($.10 x
80 checks), for a total of $18 per month or $216 per C. Assets that are considered obsolete that maintain
year. The premium account has no check charges, a net book value.
but it will require the depositor to maintain a balance
of $2,000 more than desired. At a 10% cost of D. Decelerated depreciation.
capital, the incremental $2,000 minimum deposit will
cost $200 per year. Thus, the premium account Answer (A) is correct. An imputed cost is one that
should be selected because it is cheaper by $16 per has to be estimated. It is a cost that exists but is not
year. formally recognized in the accounting system and is
the result of a process designed to recognize
[145] Source: CMA 0697 1-30 economic reality. An imputed cost is therefore
Franklin Inc. is a medium-size manufacturer of toys that relevant to the decision-making process. For
makes 25% of its sales to Mega Company, a major example, the stated interest on a bank loan is not an
national discount retailing firm. Mega will be requiring imputed cost because it is specifically stated and
Franklin and other suppliers to use Electronic Data requires a dollar outlay. But the cost of using retained
Interchange (EDI) for inventory replenishment and trade earnings as a source of capital is unstated and has to
payment transactions as opposed to the paper-based be imputed.
systems previously used. Franklin would consider all of the
following to be advantages of using EDI in its dealings with Answer (B) is incorrect because the cost of internally
generated funds is unstated. prepaid expenses in the quick assets and excludes
income taxes payable in the current liabilities.
Answer (C) is incorrect because the costs should be
written off. Answer (C) is correct. The acid test (quick) ratio
equals quick assets (cash, trading securities, and
Answer (D) is incorrect because understated accounts receivable) divided by current liabilities.
depreciation results in unstated costs. Quick assets total $206,500 ($27,500 cash +
$64,000 trading securities + $115,000 net accounts
[147] Source: CIA 1195 IV-36 receivable). Given current liabilities of $213,500
Which of the following financial ratios is used to assess the ($67,000 accounts payable + $54,000 current notes
liquidity of a company? payable + $70,000 income taxes payable + $22,500
other current liabilities), the quick ratio is 0.967
A. Days' Sales Outstanding. ($206,500/$213,500).
B. Total Debt to Total Assets Ratio. Answer (D) is incorrect because 0.82 includes
deferred income taxes payable in the current
C. Profit Margin on Sales. liabilities.
A. $219,000 A. $92,600
B. $244,000 B. $96,135
C. $344,000 C. $90,285
D. $469,000 D. $85,000
A. $1,200,000 B. 10%
B. $750,000 C. 12.09%
C. $600,000 D. 9.90%
Answer (D) is incorrect because the lockbox system Answer (D) is incorrect because the effective rate is
will result in an additional 8 days of savings, not 3. greater than the contract rate. The usable funds are
less than the face amount of the note.
[Fact Pattern #6]
Morton Company needs to pay a supplier's invoice of [160] Source: Publisher
$60,000 and wants to take a cash discount of 2/10, net 40. (Refers to Fact Pattern #6)
The firm can borrow the money for 30 days at 11% per If Morton fails to take the discount and pays on the 40th
annum with a 9% compensating balance. Assume a day, what effective rate of annual interest is it paying the
360-day year. vendor?
A. $60,000 D. 36.73%
Answer (A) is incorrect because $60,000 is the Answer (C) is correct. By failing to take the discount,
invoice amount. the company is essentially borrowing $58,800 for 30
days. Thus, at a cost of $1,200, the company
Answer (B) is incorrect because $65,934 assumes acquires the use of $58,800, resulting in a rate of
the amount paid to the supplier is $60,000. 2.04081% ($1,200 ・$58,800) for 30 days.
Assuming a 360-day year, the effective annual rate is
Answer (C) is correct. The company will need 24.489% [2.04081% x (360 days ・30 days)].
$58,800 (98% x $60,000) to pay the invoice. In
addition, it will need a compensating balance equal to Answer (D) is incorrect because 36.73% assumes a
9% of the loan. The equation is 20-day discount period.
Answer (C) is incorrect because the prime rate must Answer (B) is incorrect because 18.36% is based on
be greater than 7.37% to make the company's terms the 40-day credit period.
preferable to those of a bank.
Answer (C) is correct. The buyer could satisfy the
Answer (D) is correct. Terms of 1/10, net 60 mean $100 obligation by paying $98 on the 10th day. By
that a buyer can save 1% of the purchase price by choosing to wait until the 40th day, the buyer is
paying 50 days early. In essence, not taking the effectively paying a $2 interest charge for the use of
discount results in the buyer's borrowing 99% of the $98 for 30 days (40-day credit period - 10-day
invoice price for 50 days at a total interest charge of discount period). The interest rate on what is
1% of the invoice price. Because a year has 7.3 essentially a 30-day loan is 2.04081% ($2 ・$98).
50-day periods (365 ・50), the credit terms 1/10, Extrapolating this 30-day rate to a yearly rate
net 60 yield an effective annualized interest charge of involves multiplying by the number of periods in a
approximately 7.37% [(1% ・99%) x 7.3]. If the year. Thus, the effective annual rate is about 24.49%
prime rate were higher than 7.37%, the buyer would [2.04081% x (360 ・30 days)].
prefer to borrow from the vendor (i.e., not pay within
the discount period) rather than from a bank. Answer (D) is incorrect because 36.72% is based on
Consequently, an 8% prime rate could cause the a 20-day credit period.
vendor's receivables to increase.
[164] Source: Publisher
[162] Source: Publisher A firm has daily cash receipts of $300,000. A bank has
A company has just borrowed $2 million from a bank. The offered to provide a lockbox service that will reduce the
stated rate of interest is 10%. If the loan is discounted and collection time by 3 days. The bank requires a monthly fee
is repayable in one year, the effective rate on the loan is of $2,000 for providing this service. If money market rates
approximately are expected to average 6% during the year, the additional
annual income (loss) of using the lockbox service is
A. 8.89%
A. ($24,000)
B. 9.09%
B. $12,000
C. 10.00%
C. $30,000
D. 11.11%
D. $54,000
Answer (A) is incorrect because the prepayment of
interest reduces the funds available, resulting in an Answer (A) is incorrect because ($24,000) ignores
effective interest rate greater than the contract rate. the additional interest revenue from investing the
increased funds.
Answer (B) is incorrect because the prepayment of
interest reduces the funds available, resulting in an Answer (B) is incorrect because $12,000 is based on
effective interest rate greater than the contract rate. 2 days of accelerated inflows rather than 3.
Answer (C) is incorrect because 10% is the contract Answer (C) is correct. Because collections will be
rate. The effective rate is higher because the full $2 accelerated by 3 days at a rate of $300,000 per day,
million face amount of the note will not be available to the company will have an additional $900,000 to
the borrower. invest. At a rate of 6%, the interest earned will be
$54,000 per year. However, the bank will charge
Answer (D) is correct. If the loan is discounted, the $24,000 (12 months x $2,000 per month) for its
borrower receives the face amount minus the prepaid services. Thus, the firm will increase its income by
interest. Thus, the borrower will receive proceeds of $30,000 ($54,000 - $24,000).
$1,800,000 [$2,000,000 - (10% x $2,000,000)].
The effective interest rate is 11.11% ($200,000 ・ Answer (D) is incorrect because $54,000 ignores the
$1,800,000). $24,000 bank service charge.
Answer (A) is incorrect because $20 million ignores Answer (C) is incorrect because $13,778 is the
the increase in current assets. This answer would be average daily sales under the new policy.
appropriate if noncurrent assets were being financed.
Answer (D) is incorrect because $98,890 is the
Answer (B) is correct. If current liabilities are $200 amount of the additional receivables.
million and the current ratio (current assets ・current
liabilities) is 2.2, current assets must be $440 million [167] Source: Publisher
(2.2 x $200 million). If X amount of commercial The following information regarding inventory policy was
paper is issued to finance inventory (current assets), assembled by the TKF Corporation. The company uses a
thereby increasing both current assets and current 50-week year in all calculations.
liabilities by X, the level of current assets at which the
new current ratio will be 2.0 is $480 million ($440 Sales 12,000 units per year
million + $40 million of commercial paper). Order quantity 4,000 units
Safety stock 1,500 units
($440 + X) ・($200 + X) = 2.0 Lead time 5 weeks
$440 + X = 2($200 + X) The reorder point is
$440 + X = $400 + 2X
$440 = $400 + X A. 5,500 units
X = $40
B. 2,700 units
Answer (C) is incorrect because $240 million is the
amount of working capital both before and after the C. 1,200 units
issuance of commercial paper.
D. 240 units
Answer (D) is incorrect because $180 million is a
nonsense answer. Answer (A) is incorrect because 5,500 units equals
the order size plus the safety stock.
[166] Source: Publisher
The following information regarding a change in credit Answer (B) is correct. The reorder point equals the
policy was assembled by the Wilson Wax Company. The inventory to be sold during the lead time plus any
company has a required rate of return of 11% and a safety stock. If the weekly sales are 240 units
variable cost ratio of 50%. The opportunity cost of a longer (12,000 ・50 weeks) and the lead time is 5 weeks,
collection period is assumed to be negligible. sales during the lead time should be 1,200 units.
Adding the 1,200 units of expected sales to the
Old Credit Policy New Credit Policy 1,500 units of safety stock produces a reorder point
----------------- ----------------- of 2,700 units.
Sales $4,600,000 $4,960,000
Average collection period 30 days 35 days Answer (C) is incorrect because 1,200 units omits
The pretax cost of carrying the additional investment in safety stock.
receivables, assuming a 360-day year, is
Answer (D) is incorrect because 240 units is the
A. $5,439 average weekly usage.
Answer (C) is incorrect because 7.69% assumes the Answer (D) is incorrect because 13.79% ignores the
interest is not paid in advance. transaction costs.
Answer (D) is correct. Assuming a $1,000 loan, the [171] Source: Publisher
interest at 6% for 1 year is $60. Hence, the proceeds Parkison Company can increase annual sales by $150,000
of the loan with discounted (paid-in-advance) interest if it sells to a new, riskier group of customers. The
are $940 ($1,000 - $60). Also, 22% of the note, or uncollectible accounts expense is expected to be 16% of
$220, cannot be used by the borrower because of sales, and collection costs will be 4%. The company's
the compensating balance requirement. manufacturing and selling expenses are 75% of sales, and
Consequently, only $720 is available for use. Paying its effective tax rate is 38%. If Parkison accepts this
$60 interest for the use of $720 results in an effective opportunity, its after-tax income will increase by
cost of borrowing of 8.33% ($60 ・$720).
A. $2,850
[169] Source: Publisher
During the year, Mason Company's current assets B. $4,650
increased by $130, current liabilities decreased by $60,
and net working capital C. $7,500
Answer (A) is incorrect because 3.45% ignores the Answer (B) is incorrect because 24.5% assumes
transaction costs and fails to annualize the terms of 2/10, net 40.
percentage.
Answer (C) is incorrect because 37.1% assumes
Answer (B) is incorrect because 3.56% fails to terms of 3/10, net 40.
annualize the result.
Answer (D) is correct. Assume that the gross amount
of an invoice is $1,000. Given a 3% discount, the C. $210,000
buyer will pay $970 on the tenth day. Thus, the seller
is forgoing $30 to receive payment 20 days sooner D. $235,000
than would otherwise be required. The 20-day
interest rate (rounded) is 3.09% ($30 ・$970). The Answer (A) is incorrect because $130,000 equals
number of 20-day periods in a year is 18 (360 ・20). the purchases for the month.
If the interest rate is 3.09% for each 20-day period,
the annual interest rate (rounded) is 55.6% (18 x Answer (B) is incorrect because $140,000 excludes
3.09). other operating expenses, payroll, and interest.
Answer (C) is incorrect because $83,333 results Answer (B) is correct. Given that collections will be
from using sales instead of cost of sales throughout accelerated by 2 days, at the rate of $300,000 per
the calculation. day, the company will have an additional $600,000
to invest. At 11%, the interest earned will be
Answer (D) is correct. Inventory turnover equals cost $66,000 per year. However, the bank will charge
of sales divided by average inventory. Given cost of $36,000 (12 months x $3,000 per month) for its
sales of $35,000,000 and a turnover rate of 10 times, services. Thus, the firm will increase its pretax income
the average inventory is $3,500,000. If the turnover by $30,000 ($66,000 - $36,000).
rate increases to 15 times, the average inventory will
decline to $2,333,333 ($35,000,000 ・15), thereby Answer (C) is incorrect because $66,000 fails to
releasing $1,166,667 of funds. At a 5% cost of subtract the $36,000 cost of the lockbox system.
capital, the pretax savings is $58,333 (5% x
$1,166,667). Answer (D) is incorrect because $63,000 assumes
the annual fee is $3,000.
Answer (B) is correct. Working capital is defined as Answer (B) is incorrect because $370 million results
the excess of current assets over current liabilities. in a current ratio of 1.25.
Refinancing short-term debt with long-term debt
Answer (C) is correct. Given that current liabilities
are $200 million and the current ratio (current assets C. $50,000
・current liabilities) is 1.70, current assets must be
$340 million ($200 million x 1.70). If X amount of D. $120,000
commercial paper is issued, thereby increasing both
current assets and current liabilities by X, and the
new current ratio is the contractual minimum of 1.40 Answer (A) is incorrect because $59,125 is the
to 1, the maximum amount of commercial paper that annual lockbox cost.
can be issued is $150 million.
Answer (B) is correct. If payments are collected 2
($340 + X) current assets ・($200 + X) current liabilities = days earlier, the company can earn $120,000
1.4 ($20,000 x 50 payments per day x 2 days x .06) at a
$340 + X = 1.4 ($200 + X) cost of $59,125 [$50,000 + (50 payments x 365
$340 + X = 280 + 1.4X days x $.50)], a gain of $60,875.
$340 = $280 + .4X
$60 = .4X Answer (C) is incorrect because $50,000 is the
X = $150 annual fixed fee.
Answer (D) is incorrect because $280 million results Answer (D) is incorrect because $120,000 is the
in a current ratio of 1.29. annual savings without regard to costs.
Answer (B) is incorrect because $40,500 is the result Answer (A) is correct. Company A will withhold
of assuming an incremental average inventory of $6,000 (6% x $100,000) as a reserve against returns
$900,000. and allowances and $1,400 (1.4% x $100,000) as a
commission. The remaining $92,600 will be reduced
Answer (C) is incorrect because $64,800 is the by interest at the rate of 15% annually. The interest
interest cost for 12 months. charge will be $2,315, assuming a 360-day year
[($92,600 x .15) ・(60-day payment period ・360
Answer (D) is incorrect because $81,000 is the result days)]. The proceeds to be received by Gatsby equal
of assuming an incremental average inventory of $90,285 ($92,600 - $2,315).
$900,000 and a 12-month period.
Answer (B) is incorrect because Company B will
[187] Source: CMA Samp Q1-7 produce proceeds of only $89,964.
On January 1, Scott Corporation received a $300,000 line
of credit at an interest rate of 12% from Main Street Bank Answer (C) is incorrect because Company C will
and drew down the entire amount on February 1. The line produce proceeds of only $90,190.
of credit agreement requires that an amount equal to 15%
of the loan be deposited into a compensating balance Answer (D) is incorrect because Company D will
account. What is the effective annual cost of credit for this produce proceeds of only $90,241.
loan arrangement?
[189] Source: Publisher
A. 11.00% Mason Company's board of directors has determined 4
options to increase working capital next year. Option 1 is
B. 12.00% to increase current assets by $120 and decrease current
liabilities by $50. Option 2 is to increase current assets by
C. 12.94% $180 and increase current liabilities by $30. Option 3 is to
decrease current assets by $140 and increase current
D. 14.12% liabilities by $20. Option 4 is to decrease current assets by
$100 and decrease current liabilities by $75. Which option
Answer (A) is incorrect because 11.00% is the should Mason choose to maximize net working capital?
nominal rate for 11 months.
A. Option 1.
Answer (B) is incorrect because 12.00% is the
nominal rate of interest. B. Option 2.
Answer (D) is correct. Annual interest is $36,000 Answer (A) is correct. Working capital is the excess
($300,000 x 12%), and the amount available is of current assets over current liabilities. An increase in
$255,000 [$300,000 - (15% x $300,000)]. Thus, current assets or a decrease in current liabilities will
the effective interest rate is 14.12% ($36,000 ・ increase working capital. Option 1 maximizes Mason
$255,000). Company's net working capital, increasing it by $170
($120 + $50).
[188] Source: Publisher
Gatsby, Inc. is going to begin factoring its accounts Answer (B) is incorrect because option 2 increases
receivable and has collected information on the following net working capital by $150.
four finance companies:
Answer (C) is incorrect because option 3 decreases
Annual net working capital by $160.
Required Interest
Reserves Commissions Charge Answer (D) is incorrect because option 4 decreases
-------- ----------- -------- net working capital by $25.
Company A 6% 1.4% 15%
Company B 7% 1.2% 12% [190] Source: Publisher
Company C 5% 1.7% 20% Hendrix, Inc. is interested in purchasing a $100 U.S.
Company D 8% 1.0% 5% Treasury bill and was presented with the following options:
Which company will give Gatsby the highest proceeds from
a $100,000 account due in 60 days? Assume a 360-day Due Date Discount Rate
year. -------- -------------
Option 1 180 days 6%
A. Company A. Option 2 360 days 3.5%
Option 3 120 days 8%
B. Company B. Option 4 240 days 4.5%
If Hendrix wishes to buy the Treasury bill at the lowest
C. Company C. purchasing price, which option should be chosen, assuming
a 360-day year?
D. No; present value is $7,461.
A. Option 1.
Answer (A) is incorrect because it is based on the
B. Option 2. factor for a four-payment annuity starting next year.
Answer (D) is incorrect because it substitutes the C. $30,000 will be spent on new investment.
APR for the compounded rate.
D. The dividend per share will equal $3.00.
[192] Source: Publisher
A man offers to buy your car with 4, equal annual Answer (A) is incorrect because retained earnings
payments of $3,000, beginning 2 years from today. will increase by $70,000.
Assuming you're indifferent to cash versus credit, that you
can invest at 10%, and that you want to receive $9,000 for Answer (B) is incorrect because each share will
the car, should you accept? receive a $3 dividend.
A. Yes; present value is $9,510. Answer (C) is incorrect because $70,000 is being
retained for new investment.
B. Yes; present value is $11,372.
Answer (D) is correct. Earnings per share is $10
C. No; present value is $8,645. ($100,000 ・10,000 shares). Of the $10, 30% or $3
is paid out as a dividend.
Answer (A) is correct. The difference between
[195] Source: Publisher collections and payables is $5,000 daily. Five days'
A firm has $3 million in total assets and $1.65 million in worth amounts to $25,000 of float.
equity. How much of its $500,000 capital budget should be
debt-financed to retain the same debt-equity ratio? Answer (B) is incorrect because $40,000 ignores the
impact of payables and the five-day float period.
A. $50,000
Answer (C) is incorrect because $175,000 ignores
B. $225,000 the impact of receivables.
Answer (D) is incorrect because it is not based on Answer (A) is incorrect because the lead time does
the current 45% to 55% ratio. not affect the amount of EOQ.
[196] Source: Publisher Answer (B) is correct. The lead times does not affect
What is the benefit for a firm with daily sales of $15,000 to the EOQ, it just means the order should be placed
be able to speed up collections by 2 days, assuming an 8% four days earlier.
annual opportunity cost of funds?
Answer (C) is incorrect because the lead time does
A. $2,400 daily benefit. not affect the amount of EOQ.
B. $2,400 annual benefit. Answer (D) is incorrect because the lead time does
not affect the amount of EOQ.
C. $15,000 annual benefit.
[199] Source: Publisher
D. $30,000 annual benefit. What is the economic order quantity for an auto dealer
selling 3,000 cars per year, at a cost of $700 per order,
Answer (A) is incorrect because the benefit is and a carrying cost of $125 per automobile?
annually, not daily.
A. 130 cars.
Answer (B) is correct. Speeding up collections by
two days will free up $30,000. At 8% interest, the B. 168 cars.
savings in interest costs will be $2,400 annually.
C. 183 cars.
Answer (C) is incorrect because this is the amount of
daily sales. D. 336 cars.
Answer (D) is incorrect because this is the reduction Answer (A) is incorrect because it fails to multiply the
in receivables. numerator by 2.
[197] Source: Publisher Answer (B) is incorrect because it does not include
Average daily collection of checks for a firm is $40,000. all elements in the numerator.
The firm also writes on the average $35,000 of checks
daily. If the collection period for checks is 5 days, calculate Answer (C) is correct. The formula is:
the net float.
EOQ = square root of [2(3,000)(700)] ・125
A. $25,000 = square root of 33,600
= 1,833
B. $40,000
Answer (D) is incorrect because it will not minimize
C. $175,000 the total costs of ordering and holding inventory.
Answer (A) is incorrect because it reverses the Answer (B) is correct. The net float for a firm is the
interest calculations. dollar difference between a company's bank balance
and its book balance of cash. This amount is found
Answer (B) is incorrect because a rate decline will by subtracting the average amount of checks written
increase the value of bonds. daily, $35,000, from the average amount of checks
collected daily, $40,000. This number, $5,000, is
Answer (C) is correct. The decline in rates will then multiplied by the average collection period for
increase the value of bonds already outstanding. checks, 5 days, to get a net float of $25,000.
Since the market rate initially matched the coupon
rate, the value would have been $1,000. The Answer (C) is incorrect because $175,000 is
calculation of value following the rate decline would average daily written checks of $35,000 multiplied by
be as follows using 6% present value tables. 5 days.
Answer (B) is incorrect because $1,200 is $15,000 x Answer (C) is correct. The first step in calculating
8%. HCC's net tax liability is to subtract the exclusion for
tax-exempt interest from income, for a gross income
Answer (C) is incorrect because $600 is ($15,000 ・ amount of $69,000 ($80,000 - $11,000). Next, the
2) x 8%. deprecation deduction is subtracted from gross
income, for a taxable income of $59,000 ($69,000 -
Answer (D) is incorrect because $7,500 is daily $10,000). Then, the taxable income is multiplied by
sales, $15,000, divided by two. the tax rate, for a gross tax liability of $17,700
($59,000 x 30%). Finally, net tax liability is
[202] Source: Publisher computed by subtracting the tax credits from the
Average daily collection of checks for Firm X is $40,000. gross tax liability. Therefore, HCC's net tax liability is
Firm X also writes, on average, $35,000 in checks daily. If $9,700 ($17,700 - $8,000).
the collection period for checks is five days, what is the net
float for Firm X? Answer (D) is incorrect because $2,000 is the net tax
liability found when incorrectly treating the exclusion
as a credit. Answer (A) is incorrect because warranty expenses
are not deductible until paid.
[204] Source: Publisher
(Refers to Fact Pattern #7) Answer (B) is incorrect because dividends on
Which item reduces HCC's gross tax liability by the largest common stock are never deductible by a
amount? corporation; they are distributions of after-tax
income.
A. Gross income.
Answer (C) is incorrect because amounts accrued by
B. The tax-exempt interest exclusion. an accrual-basis taxpayer to be paid to a related
cash-basis taxpayer in a subsequent period are not
C. The depreciation deduction. deductible until the latter taxpayer includes the items
in income. This rule effectively puts related taxpayers
D. The tax credit. on the cash basis.
Answer (A) is incorrect because an exclusion or Answer (D) is correct. Sec. 162(a) states that a
deduction reduces gross tax liability by the amount of deduction is allowed for the ordinary and necessary
the exclusion or deduction multiplied by the tax rate. expenses incurred during the year in any trade or
Therefore, the exclusion will reduce HCC's gross tax business. A corporation may therefore deduct a
liability by $3,300 ($11,000 x .30), and the reasonable amount for compensation. Accrued
deduction will reduce HCC's gross tax liability by vacation pay is a form of compensation that results in
$3,000 ($10,000 x .30). an allowable deduction for federal income tax
purposes.
Answer (B) is incorrect because an exclusion or
deduction reduces gross tax liability by the amount of [206] Source: CMA 1291 2-12
the exclusion or deduction multiplied by the tax rate. All of the following are adjustments/preference items to
Therefore, the exclusion will reduce HCC's gross tax corporate taxable income in calculating alternative minimum
liability by $3,300 ($11,000 x .30), and the taxable income except
deduction will reduce HCC's gross tax liability by
$3,000 ($10,000 x .30). A. All of the gain on an installment sale of real
property in excess of $150,000.
Answer (C) is incorrect because an exclusion or
deduction reduces gross tax liability by the amount of B. Mining exploration and development costs.
the exclusion or deduction multiplied by the tax rate.
Therefore, the exclusion will reduce HCC's gross tax C. A charitable contribution of appreciated property.
liability by $3,300 ($11,000 x .30), and the
deduction will reduce HCC's gross tax liability by D. Sales commission earned in the current year but
$3,000 ($10,000 x .30). paid in the following year.
Answer (D) is correct. Credits directly reduce taxes, Answer (A) is incorrect because the gain on an
whereas exclusions and deductions reduce income installment sale of real property in excess of
prior to the computation of the gross tax liability. $150,000 is an adjustment to taxable income for
Thus, the credit reduces the gross tax liability on a purposes of computing alternative minimum taxable
dollar-for-dollar basis, or $8,000. An exclusion or income.
deduction reduces gross tax liability by the amount of
the exclusion or deduction multiplied by the tax rate. Answer (B) is incorrect because mining exploration
Accordingly, the exclusion will reduce HCC's gross and development costs are adjustments to taxable
tax liability by $3,300 ($11,000 x 30%), and the income for purposes of computing alternative
deduction will reduce HCC's gross tax liability by minimum taxable income.
$3,000 ($10,000 x 30%). Gross income does not
reduce the gross tax liability; rather, it increases the Answer (C) is incorrect because a charitable
gross tax liability by $24,000 ($80,000 x 30%). contribution of appreciated property is an adjustment
to taxable income for purposes of computing
[205] Source: CMA 1291 2-11 alternative minimum taxable income.
None of the following items are deductible in calculating
taxable income except Answer (D) is correct. Taxable income is adjusted to
arrive at alternative minimum taxable income. Some
of the common adjustments include gains or losses
A. Estimated liabilities for product warranties from long-term contracts, gains on installment sales of
expected to be incurred in the future. real property, mining exploration and development
costs, charitable contributions of appreciated
B. Dividends on common stock declared but not property, accelerated depreciation, the accumulated
payable until next year. current earnings adjustment, and tax-exempt interest
on private activity bonds issued after August 7, 1986.
C. Bonus accrued but not paid by the end of the year A sales commission accrued in the current year but
to a cash-basis 90% shareholder. paid in the following year is not an example of an
AMT adjustment.
D. Vacation pay accrued on an
employee-by-employee basis. [207] Source: Publisher
The deferral or nonrecognition of gains is not allowed for
tax purposes when the transaction is a(n)